CAT Previous Year Paper 2003

CAT 2003  

Data Interpretation 

Instructions 

DIRECTIONS for the following three questions: In each question, there are two statements: A and B, either of which can be true or false on the basis of the information given below. 

A research agency collected the following data regarding the admission process of a reputed management school in India.

Q. 1 Statement A: The success rate of moving from written test to interview stage for males was worse than for females in 2003. 

 

Statement B: The success rate of moving from written test to interview stage for females was better in 2002 than in 2003. 

A Only Statement A is true. 

B Only Statement B is true. 

C Both the Statements are true. 

D Neither of the two Statements is true. 

Answer: D 

Explanation: 

The success rate of moving from written test to interview stage for males in 2003 can be given by 637/60133 and for females in the same year can be given by 399/40763 . We can see that the rate of males is clearly more that that of females. hence statement A is false. 

Now the success rate of moving from written test to interview stage for females in 2002 was 138/15389 and for females in the year 2003 was 399/40763.So the rate in 2002 is less than that in 2003. . Hence both statements are false. 

Q. 2 Statement A: In 2002, the number of females selected for the course as a proportion of the number of females who bought application forms, was higher than the corresponding proportion for males. 

 

Statement B: In 2002, among those called for interview, males had a greater success rate than females. 

A Only Statement A is true. 

B Only Statement B is true. 

C Both the Statements are true. 

D Neither of the two Statements is true. 

Answer: D 

Explanation: 

In 2002, the number of females selected for the course as a proportion of the number of females who bought application forms was 48/19236 and for males was 171/61205. So rate for males was higher than that of female. Hence option A is false. 

In 2002, among those called for interview, the success rate for females was 48/138 and for males 171/684. So the rate was higher for females .Both are false. Hence option D. 

 

Q. 3 Statement A: The percentage of absentees in the written test among females decreased from 2002 to 2003. 

 

Statement B: The percentage of absentees in the written test among males was larger than among females in 2003. 

A Only Statement A is true. 

B Only Statement B is true. 

C Both the Statements are true. 

D Neither of the two Statements is true. 

Answer: A 

Explanation: 

The percentage of absentees in the written test among females in 2002 was 3847/19236 and in 2003 was 4529/45292. Thus percentage of absentees decreased from 02 to 03. Hence statement A is true. 

The percentage of absentees in the written test among males in 2003 is 3065/63298 which is clearly less than that of female in 2003. Hence statement b is false . Hence option A. 

 

Instructions 

DIRECTIONS for the following three questions: Answer the questions on the basis of the information given below. The length of an infant is one of the measures of his/her development in the early stages of his/her life. The figure below shows the growth chart of four infants in the first five months of life. 

Q. 4 After which month did Seeta’s rate of growth start to decline? 

A Second month 

B Third month 

C Fourth month 

D Never 

Answer: B 

Explanation: 

The rate of growth is the slope of the line. In the second and third months, there is an increase in the slope of the line. After the third month, there is a fall in the slope of the line i.e., the rate of growth is declining. Therefore, option b) is the correct answer. 

Q. 5 Who grew at the fastest rate in the first two months of life? 

A Geeta 

B Seeta 

C Ram 

D Shyam 

Answer: A 

Explanation: 

The rate of growth is the slope of the line. 

From the graphs, we can see that the slope of the line is maximum for Geeta (red line) in the first two months. So, option a) is the correct answer. 

Q. 6 The rate of growth during the third month was the lowest for 

A Geeta 

B Seeta 

C Ram 

D Shyam 

Answer: A 

Explanation: 

The rate of growth is the slope of the line. During the third month, Geeta has a negative slope whereas all the others have a positive slope. So, the rate of growth of Geeta is the least. Option a) is the correct answer. 

Q. 7 Among the four infants, who grew the least in the first five months of life? (in %) 

A Geeta 

B Seeta 

C Ram 

D Shyam 

Answer: B 

Explanation: 

Seeta’s growth percentage = (57-50)/50 * 100% = 14% 

Geeta’s growth percentage = (61-49)/49 * 100% = 24.5% 

Ram’s growth percentage = (61-52)/52 * 100% = 17.30% 

Shyam’s growth percentage = (61 – 53.5)/53.5 * 100% = 14.01% 

So, Seeta’s growth percentage is the least. 

 

Instructions 

DIRECTIONS for the following three questions: Answer the questions on the basis of the information given below. 

The table below provides certain demographic details of 30 respondents who were part of a survey. The demographic characteristics are: gender, number of children, and age of respondents. The first number in each cell is the number of respondents in that group. The minimum and maximum age of respondents in each group is given in brackets. For example, there are five female respondents with no children and among these five, the youngest is 34 years old, while the oldest is 49. 

Q. 8 The percentage of respondents aged less than 40 years is at least 

A 10% 

B 16.67% 

C 20.0% 

D 30% 

Answer: D 

Explanation: 

The number of people with age less than 40 are at least: 

In no of children 0 category => 1 male and 1 female 

In no of children 1 category => 1 male and 1 female 

In no of children 2 category => 1 male and 1 female 

In no of children 3 category => 2 male and 1 female 

There are at least 9 people who are less than 40. 

9 people out of 30 people => 30% 

Q. 9 Given the information above, the percentage of respondents older than 35 can be at most 

A 30% 

B 73.33% 

C 76.67% 

D 90% 

Answer: C 

Explanation: 

There are at most 23 people with age greater than 35 . Hence (23/30)*100 = 76.67 . So option C. 

Q. 10 The percentage of respondents that fall into the 35 to 40 years age group (both inclusive) is at least 

A 6.67% 

B 10% 

C 13.33% 

D 26.67% 

Answer: C 

Explanation: 

There are at least 4 respondents that fall into the 35 to 40 years age group (both inclusive) . So 400/30 = 13.333 % . Hence , the percentage of respondents that fall into the 35 to 40 years age group (both inclusive) is at least 13.333%. 

 

Instructions 

DIRECTIONS for the following three questions: Answer the questions on the basis of the information given below. 

Spam that enters our electronic mailboxes can be classified under several spam heads. The following table shows the distribution of such spam worldwide over time. The total number of spam emails received during December 2002 was larger than the number received in June 2003. The total number of spam emails received during September 2002 was larger than the number received in March 2003. The figures in the table represent the percentage of all spam emails received during that period, falling into those respective categories. 

 

Q. 11 In which category was the percentage of spam emails increasing but at a decreasing rate? 

A Financial 

B Scams 

C Products 

D None of the above 

Answer: C 

Explanation: 

It is clear from the table that for the products category the percentage of spam mail is increasing from sep02 to June 03 . But the rate at which it is growing is decreasing. Hence option c . 

 

Q. 12 In the health category, the number of spam emails received in December 2002 as compared to June 2003. 

A was larger 

B was smaller 

C was equal 

D cannot be determined 

Answer: A 

Explanation: 

In dec 2002 , 19 % of all spam mails were from the health category and In jun 2003 , 18 % of all spam mails were from the health category. Now since the total spam mails received in dec 02 were greater than that in jun 03 . Number of spam mails in the health category in dec 02 will be greater than that in jun 03. 

 

Q. 13 In the financial category, the number of spam emails received in September 2002 as compared to March 2003. 

A was larger 

B was smaller 

C was equal 

D cannot be determined 

Answer: D 

Explanation: 

Since we don’t know by how much more total spam mails were received in Sep 02 as compared to mar 03 we can’t compare the number of spam emails in the financial category received in September 2002 as to that in March 2003. Hence option D. 

 

Instructions 

DIRECTIONS for the following three questions: Answer the questions on the basis of the information given below. 

One of the functions of the Reserve Bank of India is to mobilize funds for the Government of India by issuing securities. The following table shows details of funds mobilized during the period July 2002 – July 2003. Notice that on each date there were two rounds of issues, each with a different maturity. 

Q. 14 How many times was the issue of securities under-subscribed, i.e., how often did the total amount mobilized fall short of the amount notified? 

A 0 

B 1 

C 2 

D 3 

Answer: B 

Explanation: 

Only one security issued on 17 July 2002 was undersubscribed. 

Q. 15 Which of the following is true? 

A The second round issues have a higher maturity than the first round for all dates. 

B The second round issue of any date has a lower maturity only when the first round notified amount exceeds that of the second round. 

C On at least one occasion, the second round issue having lower maturity received a higher number of competitive bids. 

D None of the above three statements is true. 

Answer: C 

Explanation: 

The security issued in June 2003 had lower maturity in 2nd round but no. of bids were more . Hence option C. 

 

Q. 16 Which of the following statements is NOT true? 

A Competitive bids received always exceed non-competitive bids received. 

B The number of competitive bids accepted does not always exceed the number of non-competitive bids accepted. 

C The value of competitive bids accepted on any particular date is never higher for higher maturity. 

D The value of non-competitive bids accepted in the first round is always greater than that in the second round. 

Answer: D 

Explanation: 

For the security issued in 4th June 2003 , the value of non-competitive bids accepted in the first round islesas than that in the second round Hence option D is not true. 

 

Instructions 

DIRECTIONS for the following three questions: Answer the questions on the basis of the information given below.

Each point in the graph below shows the profit and turnover data for a company. Each company belongs to one of the three industries: textile, cement and steel. 

 

Q. 17 For how many companies does the profit exceed 10% of turnover? 

A 8 

B 7 

C 6 

D 5 

Answer: B 

Explanation: 

There are in total 7 companies – 3 steels , 2 cement , 2 textile for which the profit is more than 10% of the turnover. 

 

Q. 18 For how many steel companies with a turnover of more than 2000 is the profit less than 300? 

A 0 

B 1 

C 2 

D 7 

Answer: C 

Explanation: 

There are 2 steel companies with turnover of more than 2000 and having profit less than 300. 

 

Q. 19 An investor wants to buy stock of only steel or cement companies with a turnover more than 1000 and profit exceeding 10% of turnover. How many choices are available to the investor? 

A 4 

B 5 

C 6 

D 7 

Answer: B 

Explanation: 

There are 3 steel and 2 cement companies with a turnover more than 1000 and profit exceeding 10% of turnover. hence there are 5 choices for the investor. 

Instructions 

DIRECTIONS for the following three questions: Answer the questions on the basis of the information given below. Details of the top 20 MBA schools in the US as ranked by US News and World Report, 1997 are given below. 

 

Q. 20 Madhu has received admission in all schools listed above. She wishes to select the highest overall ranked school whose annual tuition fee does not exceed $23,000 and median starting salary is at least $70,000. Which school will she select? 

A University of Virginia. 

B University of Pennsylvania 

C Northwestern University 

D University of California – Berkeley 

Answer: D 

Explanation: 

From table it is clear that for University of California – Berkeley annual tuition fee does not exceed $23,000 and median starting salary is at least $70,000. Hence option D. 

 

Q. 21 In terms of staring salary and tuition fee, how many schools are uniformly better (higher median starting salary AND lower tuition fee) than Dartmouth College? 

A 1 

B 2 

C 3 

D 4 

Answer: B 

Explanation: 

There are only 2 colleges – New York university and stanford university which are uniformly better (higher median starting salary AND lower tuition fee) than Dartmouth College. 

 

Q. 22 How many schools in the list above have single digit rankings on at least 3 of the 4 parameters (overall ranking, ranking by academics, ranking by recruiters and ranking by placement)? 

A 10 

B 5 

C 7 

D 8 

Answer: D 

Explanation: 

We can clearly make out from the given table that there are 8 schools in the list which have single digit rankings on at least 3 of the 4 parameters. 

 

Instructions 

DIRECTIONS for the following three questions: Answer the questions on the basis of the information given below.

Table A below provides data about ages of children in a school. For the age given in the first column, the second column gives the number of children not exceeding the age. For example, first entry indicates that there are 9 children aged 4 years or less. Tables B and C provide data on the heights and weights respectively of the same group of children in a similar format. Assuming that an older child is always taller and weighs more than a younger child, answer the following questions. 

Q. 23 What is the number of children of age 9 years of less whose height does not exceed 135 cm? 

A 48 

B 45 

C 3 

D Cannot be determined 

Answer: B 

Explanation: 

Number of children of age 9 years or less are 48 and those whose height does not exceed 135 cm are 45 . 45 is lesser, hence the answer is 45. 

Q. 24 How many children of age more than 10 years are taller than 150 cm and do not weigh more than 48 kg? 

A 16 

B 40 

C 9 

D Cannot be determined 

Answer: A 

Explanation: 

There are 40 children of age more than 10 years and 25 children that are taller than 150 cm. Considering 25 which is less than 40 . Also there are 9 children whose weight is more than 48 kg. Hence there are 25-9 = 16 children whose weight will be less than 48 kg and all other requirements. 

Q. 25 Among the children older than 6 years but not exceeding 12 years, how many weigh more than 38 kg.? 

A 34 

B 52 

C 44 

D Cannot be determined 

Answer: C 

Explanation: 

Number of children older than 6 years but not exceeding 12 years are 55 and number older than 12 years are 23 . Also children weighing more than 38 kgs are 67. Out of these 67 , 23 will have age more than 12 . Hence we have 67 – 23 = 44 childrens will the given requirement. Hence option C. 

 

Instructions 

DIRECTIONS for the following two questions: Answer the questions on the basis of the information given below.

An industry comprises four firms (A, B, C, and D). Financial details of these firms and of the industry as a whole for a particular year are given below. Profitability of a firm is defined as profit as a percentage of sales. 

Q. 26 Which firm has the highest profitability? 

A A 

B B 

C C 

D D 

Answer: D 

Explanation: 

From the table we can see that , profitability in A , B ,C , D (4914)/(24568) , (4075)/(25468) , (4750)/(23752) , (3946)/(15782) . So clearly D has the highest profitability. 

 

Q. 27 If firm A acquires firm B, approximately what percentage of the total market (total sales) will they corner together? 

A 55% 

B 45% 

C 35% 

D 50% 

Answer: A 

Explanation: 

If A acquires firm B , the total sales of 2 firms is 50036 . now the percent market share can be given by 50036*100/89576 which is equal to around 55% . Hence option A. 

 

Instructions 

DIRECTIONS for the following three questions: Answer the questions on the basis of the information given below. 

A, B, C, D, E, and F are a group of friends. There are two housewives, one professor, one engineer, one accountant and one lawyer in the group. There are only two married couples in the group. The lawyer is married to D, who is a housewife. No woman in the group is either an engineer or an accountant. C, the accountant, is married to F, who is a professor. A is married to a housewife. E is not a housewife. 

Q. 28 Which of the following is one of the married couples? 

A A & B 

B B & E 

C D & E 

D A & D 

Answer: D 

Explanation: 

According to given conditions, we are able to infer following relations 

So A and D are married couples. 

Q. 29 What is E’s profession? 

A Engineer 

B Lawyer 

C Professor 

D Accountant 

Answer: A 

Explanation: 

According to given conditions we are able to infer, 

Hence E is an engineer. 

Q. 30 How many members of the group are males? 

A 2 

B 3 

C 4 

D Cannot be determined 

Answer: B 

Explanation: 

According to given conditions we are able to infer, 

Thus, there are 3 males. 

Instructions 

DIRECTIONS for the following two questions: Answer the questions on the basis of the information given below. 

The Head of a newly formed government desires to appoint five of the six elected members A, B, C, D, E and F to portfolios of Home, Power, Defence, Telecom and Finance. F does not want any portfolio if D gets one of the five. C wants either Home or Finance or no portfolio. B says that if D gets either Power or Telecom then she must get the other one. E insists on a portfolio if A gets one. 

Q. 31 Which is a valid assignment? 

A A-Home, B-Power, C-Defence, D-Telecom, E-Finance. 

B C-Home, D-Power, A-Defence, B-Telecom, E-Finance. 

C A-Home, B-Power, E-Defence, D-Telecom, F-Finance. 

D B-Home, F-Power, E-Defence, C-Telecom, A-Finance. 

Answer: B 

Explanation: 

Since C wants either home or finance or none so options A and D are eliminated. 

Since F does not want any portfolio if D gets one, Option C is eliminated. 

Q. 32 If A gets Home and C gets Finance, then which is NOT a valid assignment of Defense and Telecom? 

A D-Defence, B-Telecom. 

B F-Defence, B-Telecom. 

C B-Defence, E-Telecom. 

D B-Defence, D-Telecom. 

Answer: D 

Explanation: 

B says that if D gets power or telecom then he must get the other one.Option D clearly violates that. Instructions 

DIRECTIONS for the following three questions: Answer the questions on the basis of the information given below.

Rang Barsey Paint Company (RBPC) is in the business of manufacturing paints. RBPC buys RED, YELLOW, WHITE, ORANGE, and PINK paints. ORANGE paint can be also produced by mixing RED and YELLOW paints in equal proportions. Similarly, PINK paint can also be produced by mixing equal amounts of RED and WHITE paints. Among other paints, RBPC sells CREAM paint, (formed by mixing WHITE and YELLOW in the ratio 70:30) AVOCADO paint (formed by mixing equal amounts of ORANGE and PINK paint) and WASHEDORANGE paint (formed by mixing equal amounts of ORANGE and WHITE paint). The following table provides the price at which RBPC buys paints. 

Q. 33 The cheapest way to manufacture AVOCADO paint would cost 

A Rs. 19.50 per litre. 

B Rs. 19.75 per litre 

C Rs. 20.00 per litre. 

D Rs. 20.25 per litre. 

Answer: B 

Explanation: 

AVOCADO paint can be manufactured by adding orange and pink in equal quantity. 0.5 ltr of orange would cost 11 and the cheapest way to make pink would be by mixing white and red , so the cost for 0.5 ltr of pink comes out to be 8.75 . SO total cost becomes 11+8.75 = 19.75 Rs. which is the cheapest. 

Q. 34 WASHEDORANGE can be manufactured by mixing 

A CREAM and RED in the ratio 14:10. 

B CREAM and RED in the ratio 3:1. 

C YELLOW and PINK in the ratio 1:1. 

D RED, YELLOW, and WHITE in the ratio 1:1:2. 

Answer: D 

Explanation: 

WASHEDORANGE can be made by mixing orange and white , orange can be made by mixing equal quantities red and yellow which would be 1/2 of white quantity. Thus RED, YELLOW, and WHITE in the ratio 1:1:2 are needed. 

Q. 35 Assume that AVOCADO, CREAM and WASHEDORANGE each sells for the same price. Which of the three is the most profitable to manufacture? 

A AVOCADO 

B CREAM 

C WASHEDORANGE 

D Sufficient data is not available. 

Answer: B 

Explanation: 

Cream would undoubtedly be the most profitable as the maximum amount of white paint is used in it and white is the cheapest out of all other paints. Hence option B. 

 

Instructions 

DIRECTIONS for the following three questions: Answer the questions on the basis of the information given below. 

Seven varsity basketball players (A, B, C, D, E, F, and G) are to be honoured at a special luncheon. The players will be seated on the dais in a row. A and G have to leave the luncheon early and so must be seated at the extreme right. B will receive the most valuable player’s trophy and so must be in the centre to facilitate presentation. C and D are bitter rivals and therefore must be seated as far apart as possible. 

Q. 36 Which of the following cannot be seated at either end? 

A C 

B D 

C F 

D G 

Answer: C 

Explanation: 

According to given conditions we can conclude , 

Hence , F cannot be seated at either end. 

 

Q. 37 Which of the following pairs cannot be seated together? 

A B & D 

B C & F 

C D & G 

D E & A 

Answer: D 

Explanation: 

According to given conditions following arrangements are possible, 

Hence E and A cannot be seated together. 

 

Q. 38 Which of the following pairs cannot occupy the seats on either side of B? 

A F & D 

B D & E 

C E & G 

D C & F 

Answer: C 

Explanation: 

According to given conditions possible arrangement is as, 

Hence E and G cannot occupy seats on either sides of B . 

 

Instructions 

DIRECTIONS for the following two questions: Answer the questions on the basis of the information given below. 

Some children were taking free throws at the basketball court in school during lunch break. Below are some facts about how many baskets these children shot. 

i. Ganesh shot 8 baskets less than Ashish. 

ii. Dhanraj and Ramesh together shot 37 baskets. 

iii. Jugraj shot 8 baskets more than Dhanraj. 

iv. Ashish shot 5 baskets more than Dhanraj. 

v. Ashish and Ganesh together shot 40 baskets. 

 

Q. 39 Which of the following statements is true? 

A Dhanraj and Jugraj together shot 46 baskets. 

B Ganesh shot 18 baskets and Ramesh shot 21 baskets. 

C Dhanraj shot 3 more baskets than Ramesh. 

D Ramesh and Jugraj together shot 29 baskets. 

Answer: A 

Explanation: 

Let a,d,j,g be the shots put by ashish,dhanraj,ganesh and jugraj respectively. According to given conditions we have , g=a-8; 

d+r=37; 

j=d+8; 

a=5+d; 

a+g=40 . 

Solving we have a=24, d=19 and j=27.so d+j=45 

 

Instructions 

DIRECTIONS for the following three questions: Answer the questions on the basis of the information given below. 

Five women decided to go shopping in M.G. Road, Bangalore. They arrived at the designated meeting place in the following order: 

1. Archana, 

2. Chellamma, 

3. Dhenuka, 

4. Helen, and 

5. Shahnaz. 

Each woman spent at least Rs. 1000. Below are some additional facts about how much they spent during their shopping spree. 

i. The woman who spent Rs. 2234 arrived before the lady who spent Rs. 1193. 

ii. One woman spent Rs. 1340 and she was not Dhenuka. 

iii. One woman spent Rs. 1378 more than Chellamma. 

iv. One woman spent Rs. 2517 and she was not Archana. 

v. Helen spent more than Dhenuka. 

vi. Shahnaz spent the largest amount and Chellamma the smallest. 

 

Q. 40 What was the amount spent by Helen? 

A Rs. 1193 

B Rs. 1340 

C Rs. 2234 

D Rs. 2517 

Answer: B 

Explanation: 

From the given statements, we can infer that the amounts spent by the women are Rs.2234, Rs.1193, Rs.1340, and Rs.2517. Also, we know that one of the 5 women spent Rs.1378 more than Chellamma. Also, we know that each woman spent at least Rs.1000. 

The person who spent Rs.2234 cannot be the person who spent Rs.1378 more than Chellamma. Also, we know that Chellamma spent the least among the five women. 

Case 1: 

Chellamma spent Rs.1193. 

=> One of the 5 women spent 1193+1378 = Rs.2571. 

We know that Shahnaz spent the most. Therefore, Shahnaz should have spent Rs.2571. 

Archana didn’t spend Rs.2517. Also, Helen spent more than Dhenuka. 

Therefore, Helen should have spent Rs.2517. 

Dhenuka didn’t spend Rs.1340. 

Therefore, Dhenuka should have spent Rs. 2234 and Archana should have spent Rs.1340. 

However, it has been given that the woman who spent Rs. 2234 arrived before the lady who spent Rs. 1193. According to the given order, Archana arrived before Dhenuka. Therefore, we can eliminate this case. 

Case 2: 

Rs.2517 is the highest amount spent. Shahnaz spent Rs.2517. 

=> Amount spent by Chellamma = 2517 – 1378 = Rs.1139 

Dhenuka didn’t spend Rs.1340. Helen spent more than Dhenuka. Therefore, Dhenuka should have spent Rs.1193. 

Now, we know that the woman who spent Rs. 2234 arrived before the lady who spent Rs. 1193. Helen did not arrive before Dhenuka but Archana did. Therefore, Archana should have spent Rs.2234 and Helen should have spent Rs.1340. 

According to given conditions amount spent by everyone is, 

Hence, option B is the right answer. 

 

Q. 41 Which of the following amounts was spent by one of them? 

A Rs. 1139 

B Rs. 1378 

C Rs. 2571 

D Rs. 2718 

Answer: A 

Explanation: 

From the given statements, we can infer that the amounts spent by the women are Rs.2234, Rs.1193, Rs.1340, and Rs.2517. Also, we know that one of the 5 women spent Rs.1378 more than Chellamma. Also, we know that each woman spent at least Rs.1000. 

The person who spent Rs.2234 cannot be the person who spent Rs.1378 more than Chellamma. Also, we know that Chellamma spent the least among the five women. 

Case 1: 

Chellamma spent Rs.1193. 

=> One of the 5 women spent 1193+1378 = Rs.2571. 

We know that Shahnaz spent the most. Therefore, Shahnaz should have spent Rs.2571. 

Archana didn’t spend Rs.2517. Also, Helen spent more than Dhenuka. 

Therefore, Helen should have spent Rs.2517. 

Dhenuka didn’t spend Rs.1340. 

Therefore, Dhenuka should have spent Rs. 2234 and Archana should have spent Rs.1340. 

However, it has been given that the woman who spent Rs. 2234 arrived before the lady who spent Rs. 1193. According to the given order, Archana arrived before Dhenuka. Therefore, we can eliminate this case. 

Case 2: 

Rs.2517 is the highest amount spent. Shahnaz spent Rs.2517. 

=> Amount spent by Chellamma = 2517 – 1378 = Rs.1139 

Dhenuka didn’t spend Rs.1340. Helen spent more than Dhenuka. Therefore, Dhenuka should have spent Rs.1193. 

Now, we know that the woman who spent Rs. 2234 arrived before the lady who spent Rs. 1193. Helen did not arrive before Dhenuka but Archana did. Therefore, Archana should have spent Rs.2234 and Helen should have spent Rs.1340. 

According to given conditions amount spent by everyone is, 

Hence, option A is the right answer. 

 

Q. 42 The woman who spent Rs. 1193 is 

A Archana 

B Chellamma 

C Dhenuka 

D Helen 

Answer: C 

Explanation: 

From the given statements, we can infer that the amounts spent by the women are Rs.2234, Rs.1193, Rs.1340, and Rs.2517. Also, we know that one of the 5 women spent Rs.1378 more than Chellamma. Also, we know that each woman spent at least Rs.1000. 

The person who spent Rs.2234 cannot be the person who spent Rs.1378 more than Chellamma. Also, we know that Chellamma spent the least among the five women. 

Case 1: 

Chellamma spent Rs.1193. 

=> One of the 5 women spent 1193+1378 = Rs.2571. 

We know that Shahnaz spent the most. Therefore, Shahnaz should have spent Rs.2571. 

Archana didn’t spend Rs.2517. Also, Helen spent more than Dhenuka. 

Therefore, Helen should have spent Rs.2517. 

Dhenuka didn’t spend Rs.1340. 

Therefore, Dhenuka should have spent Rs. 2234 and Archana should have spent Rs.1340. 

However, it has been given that the woman who spent Rs. 2234 arrived before the lady who spent Rs. 1193. According to the given order, Archana arrived before Dhenuka. Therefore, we can eliminate this case. 

Case 2: 

Rs.2517 is the highest amount spent. Shahnaz spent Rs.2517. 

=> Amount spent by Chellamma = 2517 – 1378 = Rs.1139 

Dhenuka didn’t spend Rs.1340. Helen spent more than Dhenuka. Therefore, Dhenuka should have spent Rs.1193. 

Now, we know that the woman who spent Rs. 2234 arrived before the lady who spent Rs. 1193. Helen did not arrive before Dhenuka but Archana did. Therefore, Archana should have spent Rs.2234 and Helen should have spent Rs.1340. 

According to given conditions amount spent by everyone is, 

Hence, option C is the right answer. 

Instructions 

DIRECTIONS for the following three questions: Answer the questions on the basis of the information given below. 

Five friends meet every morning at Sree Sagar restaurant for an idli-vada breakfast. Each consumes a different number of idlis and vadas. The number of idlis consumed are 1, 4, 5, 6, and 8, while the number of vadas consumed are 0, 1, 2, 4, and 6. Below are some more facts about who eats what and how much. 

i. The number of vadas eaten by Ignesh is three times the number of vadas consumed by the person who eats four idlis. ii. Three persons, including the one who eats four vadas eat without chutney. 

iii. Sandeep does not take any chutney. 

iv. The one who eats one idli a day does not eat any vadas or chutney. Further, he is not Mukesh. v. Daljit eats idli with chutney and also eats vada. 

vi. Mukesh, who does not take chutney, eats half as many vadas as the person who eats twice as many idlis as he does. vii. Bimal eats two more idlis than Ignesh, but Ignesh eats two more vadas than Bimal. 

 

Q. 43 Which one of the following statements is true? 

A Daljit eats 5 idlis 

B Ignesh eats 8 idlis 

C Bimal eats 1 idli. 

D Bimal eats 6 idlis. 

Answer: A 

Explanation: 

According to given conditions , we get 

Hence , diljit eats 5 idlis. 

Q. 44 Which of the following statements is true? 

A Sandeep eats 2 vadas. 

B Mukesh eats 4 vadas. 

C Ignesh eats 6 vadas. 

D Bimal eats 2 vadas. 

Answer: C 

Explanation: 

According to given conditions , we get 

Hence, ignesh eats 6 wadas. 

 

Q. 45 Which of the following statements is true? 

A Mukesh eats 8 idlis and 4 vadas but no chutney. 

B The person who eats 5 idlis and 1 vada does not take chutney. 

C The person who eats equal number of vadas and idlis also takes chutney. 

D The person who eats 4 idlis and 2 vadas also takes chutney. 

Answer: C 

Explanation: 

According to given conditions , we get 

Hence ignesh is the person who eats equal number of vadas and idlis also takes chutney. 

Instructions 

DIRECTIONS for the following two questions: Answer the questions on the basis of the information given below. 

A certain perfume is available at a duty-free shop at the Bangkok international airport. It is priced in the Thai currency Baht but other currencies are also acceptable. In particular, the shop accepts Euro and US Dollar at the following rates of exchange: 

US Dollar 1 = 41 Bahts 

Euro 1= 46 Bahts 

The perfume is priced at 520 Bahts per bottle. After one bottle is purchased, subsequent bottles are available at a discount of 30%. Three friends S, R and M together purchase three bottles of the perfume, agreeing to share the cost equally. R pays 2 Euros. M pays 4 Euros and 27 Thai Bahts and S pays the remaining amount in US Dollars. 

Q. 46 How much does R owe to S in Thai Baht? 

A 428 

B 416 

C 334 

D 324 

Answer: D 

Explanation: 

Total to be paid = 1248 Baht 

Each has to pay 1248/3 = 416 Baht 

R paid 92 Baht 

M paid 184+27 = 211 Baht 

So, R owes S 416 – 92 = 324 Baht 

 

Q. 47 How much does M owe to S in US Dollars? 

A 3 

B 4 

C 5 

D 6 

Answer: C 

Explanation: 

Total to be paid = 1248 Baht 

Each has to pay 1248/3 = 416 Baht 

R paid 92 Baht 

M paid 184+27 = 211 Baht 

So, R owes S 416 – 92 = 324 Baht 

M owes S 416-211 Baht = 205 Baht = 5 US Dollars 

 

Instructions 

DIRECTIONS for the following two questions: Answer the questions on the basis of the information given below. 

New Age Consultants have three consultants Gyani, Medha and Buddhi. The sum of the number of projects handled by Gyani and Buddhi individually is equal to the number of projects in which Medha is involved. All three consultants are involved together in 6 projects. Gyani works with Medha in 14 projects. Buddhi has 2 projects with Medha but without Gyani, and 3 projects with Gyani but without Medha. The total number of projects for New Age Consultants is one less than twice the number of projects in which more than one consultant is involved. 

Q. 48 What is the number of projects in which Gyani alone is involved? 

A Uniquely equal to zero. 

B Uniquely equal to 1. 

C Uniquely equal to 4. 

D Cannot be determined uniquely. 

Answer: D 

Explanation: 

The total number of projects = 2(3+6+8+2) – 1 = 38 – 1 = 37 

So, 19 + 2(x+y) – 16 = 37 

=> x+y = 17 

The number of projects in which Medha alone is involved is 17-16 = 1 

But the number of projects in which Gyani alone is involved cannot be uniquely determined 

Q. 49 What is the number of projects in which Medha alone is involved? 

A Uniquely equal to zero. 

B Uniquely equal to 1. 

C Uniquely equal to 4. 

D Cannot be determined uniquely. 

Answer: B 

Explanation: 

The total number of projects = 2(3+6+8+2) – 1 = 38 – 1 = 37 

So, 19 + 2(x+y) – 16 = 37 

=> x+y = 17 

The number of projects in which Medha alone is involved is 17-16 = 1 

But the number of projects in which Gyani alone is involved cannot be uniquely determined 

Instructions 

DIRECTIONS for the following three questions: Answer the questions on the basis of the information given below. A city has two perfectly circular and concentric ring roads, the outer ring road (OR) being twice as long as the inner ring road (IR). There are also four (straight line) chord roads from E1, the east end point of OR to N2, the north end point of IR; from N1, the north end point of OR to W2, the west end point of IR; from W1, the west end point of OR, to S2, the south end point of IR; and from S1 the south end point of OR to E2, the east end point of IR. Traffic moves at a constant speed of 30π km/hr on the OR road, 20π km/hr on the IR road, and 15√5 km/hr on all the chord roads. 

 

Q. 50 The ratio of the sum of the lengths of all chord roads to the length of the outer ring road is

A √5 : 2

B √5 : 2π

C √5 : π

D None of the above. 

Answer: C 

Explanation: 

Let the radius of outer circle be 2R and the centre of both the circles be O. 

Triangle ON2E1 and all the other 3 similar triangles form a right angle at the centre . SO using pythagoras theorem the value of chords come out to be √5* R so total length of the chords 4 *√5 * R And circumference is equal to 2 *Pi*2R . Ratio gives option C. 

Quant 

Instructions 

For the following questions answer them individually 

Q. 51 F and M are father and mother of S, respectively. S has four uncles and three aunts. F has two siblings. The siblings of F and M are unmarried. How many brothers does M have? 

A. F has two brothers. 

B. M has five siblings. 

A The questions can be answered by one of the statements, but not by the other 

B The Q. can be answered using either of the two statements alone. 

C The questions can be answered using both the statements together, but cannot be answered using either statement alone. 

D The Q. cannot be answered even by using both the statements together. 

Answer: A 

Explanation: 

Statement B is redundant. We already know that M has 5 siblings from the question. From statement A alone, we know that F has 2 brothers and M has 2 brothers and 3 sisters. 

Q. 52 A game consists of tossing a coin successively. There is an entry fee of Rs. 10 and an additional fee of Re. 1 for each toss of coin. The game is considered to have ended normally when the coin turns heads on two consecutive throws. In this case the player is paid Rs. 100. Alternatively, the player can choose to terminate the game prematurely after any of the tosses. Ram has incurred a loss of Rs. 50 by playing this game. How many times did he toss the coin? 

A. The game ended normally. 

B. The total number of tails obtained in the game was 138. 

A The questions can be answered by one of the statements, but not by the other 

B The Q. can be answered using either of the two statements alone. 

C The questions can be answered using both the statements together, but cannot be answered using either statement alone. 

D The Q. cannot be answered even by using both the statements together. 

Answer: B 

Explanation: 

Considering statement A. The game ended normally. Here we know that last 2 results were heads and the person receives 100 Rs. and since there was overall loss of 50 Rs we can calculate no. of matches which ended in tails or non consecutive heads . hence , statement a is sufficient to answer. 

Considering statement B. The total number of tails obtained in the game was 138. So we know person lost here rs.138 and we can add any no. of single heads in between those tails such that the overall loss will be 50Rs. Hence this statement is also sufficient to answer. 

Q. 53 Each packet of SOAP costs Rs. 10. Inside each packet is a gift coupon labelled with one of the letters S, O, A and P. If a customer submits four such coupons that make up the word SOAP, the customer gets a free SOAP packets. Ms. X kept buying packet after packet of SOAP till she could get one set of coupons that formed the word SOAP. How many coupons with label P did she get in the above process? 

A. The last label obtained by her was S and the total amount spent was Rs. 210. 

B. The total number of vowels obtained was 18. 

A The questions can be answered by one of the statements, but not by the other 

B The Q. can be answered using either of the two statements alone. 

C The questions can be answered using both the statements together, but cannot be answered using either statement alone. 

D The Q. cannot be answered even by using both the statements together. 

Answer: C 

Explanation: 

Considering statements we can deduce that she got letter S in her 21st purchase. In the previous 20 purchases she had either A, O or P . Hence statement a is not enough to answer. If we consider statement b we will know that out of 20 purchases 18 are vowel so 2 purchases contained letter P . So both statements are necessary to answer the question. 

Q. 54 If A and B run a race, then A wins by 60 seconds. If B and C run the same race, then B wins by 30 seconds. Assuming that C maintains a uniform speed what is the time taken by C to finish the race? 

A. A and C run the same race and A wins by 375 metres. 

B. The length of the race is 1 km. 

A The questions can be answered by one of the statements, but not by the other 

B The Q. can be answered using either of the two statements alone. 

C The questions can be answered using both the statements together, but cannot be answered using either statement alone. 

D The Q. cannot be answered even by using both the statements together. 

Answer: C 

Explanation: 

Let the distance be x. Time taken by A is T, so time taken by B is T+60 and time taken by C is T+90. From statement A, we know that the ratio of speeds of A and C is x:(x-375) = x/T : x/(T+90) 

We have only 1 equation and 2 unknows. We need the second statement to find the values. So, the Q. can be answered by using both the statements together. 

Q. 55 The number of non-negative real roots of  2x x − 1 = 0 equals 

A 0 

B 1 

C 2 

D 3 

Answer: C 

Explanation: 

2x x − 1 = 0 for this equation only 0 and 1 i.e 2 non-negative solutions are possible. Or we can plot the graph of 2x and x+1 and determine the number of points of intersection and hence the solution. 

Q. 56 When the curves y = log10x and y = x−1  are drawn in the x-y plane, how many times do they intersect for x ≥ 1 values ? 

A Never 

B Once 

C Twice 

D More than twice 

Answer: B 

Explanation: 

Graph of log x goes on increasing in 1st quadrant and graph of 1/x goes no decreasing with both intersecting only once

Q. 57 Let A and B be two solid spheres such that the surface area of B is 300% higher than the surface area of A. The volume of A is found to be k% lower than the volume of B. The value of k must be

A 85.5

B 92.5 

C 90.5 

D 87.5 

Answer: D 

Explanation: 

Surface area of sphere A (of radius a) is 4π a2

Surface area of sphere B (of radius b) is 4π b2

=> 4π a2/4π b2 = 1/4 => a:b = 1:2 

Their volumes would be in the ratio 1:8 

Therefore, k = 7/8 * 100% = 87.5% 

Q. 58 Which one of the following conditions must p, q and r satisfy so that the following system of linear simultaneous equations has at least one solution, such that p + q + r ≠ 0? 

x+ 2y – 3z = p 

2x + 6y – 11z = q 

x – 2y + 7z = r 

A 5p -2q – r = 0 

B 5p + 2q + r = 0 

C 5p + 2q – r = 0 

D 5p – 2q + r = 0 

Answer: A 

Explanation: 

Substitute value of p,q,r in the options only option A satisfies . 

5(x+2y-3z)-2(2x+6y-11z)-(x-2y+7z) = 5x+10y-15z-4x-12y+22z-x+2y-7z = 0 

Q. 59 A leather factory produces two kinds of bags, standard and deluxe. The profit margin is Rs. 20 on a standard bag and Rs. 30 on a deluxe bag. Every bag must be processed on machine A and on Machine B. The processing times per bag on the two machines are as follows: 

The total time available on machine A is 700 hours and on machine B is 1250 hours. Among the following production plans, which one meets the machine availability constraints and maximizes the profit? 

A Standard 75 bags, Deluxe 80 bags 

B Standard 100 bags, Deluxe 60 bags 

C Standard 50 bags, Deluxe 100 bags 

D Standard 60 bags, Deluxe 90 bags 

Answer: A 

Explanation: 

Let x be no. of standard bags and y be no. of deluxe bags. According to given conditions we have 2 equations 4x+5y<=700 and 6x+10y<=1250. Here option A satisfies both the equations. 

Q. 60 The sum of 3rd and 15th elements of an arithmetic progression is equal to the sum of 6th, 11th and 13th elements of the same progression. Then which element of the series should necessarily be equal to zero? 

A 1st 

B 9th 

C 12th 

D None of the above 

Answer: C 

Explanation: 

The sum of the 3rd and 15th terms is a+2d+a+14d = 2a+16d 

The sum of the 6th, 11th and 13th terms is a+5d+a+10d+a+12d = 3a+27d 

Since the two are equal, 2a+16d = 3a+27d => a+11d = 0 

So, the 12th term is 0 

Q. 61 A test has 50 questions. A student scores 1 mark for a correct answer, -1/3 for a wrong answer, and -1/6 for not attempting a question. If the net score of a student is 32, the number of questions answered wrongly by that student cannot be less than 

A 6 

B 12 

C 3 

D 9 

Answer: C 

Explanation: 

Let the number of questions answered correctly be x and the number of questions answered wrongly be y. So, number of questions left unattempted = (50-x-y) 

So, x – y/3 – (50-x-y)/6 = 32 

=> 6x – 2y – 50 + x + y = 192 => 7x – y = 242 => y = 7x – 242 

If x = 35, y = 3 

If x = 36, y = 10 

So, min. the value of y is 3. 

The number of wrongly answered questions cannot be less than 3. 

Q. 62 Twenty-seven persons attend a party. Which one of the following statements can never be true? 

A There is a person in the party who is acquainted with all the twenty-six others. 

B Each person in the party has a different number of acquaintances. 

C There is a person in the party who has an odd number of acquaintances. 

D In the party, there is no set of three mutual acquaintances. 

Answer: B 

Explanation: 

From the options a, c and d all can possibly occur. Hence option b. Besides, if all people have different number of acquaintances, then first person will have 26 acquaintance, second person will have 25 acquaintance, third person will have 24 and so on till 27 th person will have 0 acquaintance. 0 acquaintance is practically not possible. 

Q. 63 Let g(x) = max(5 – x, x + 2). The smallest possible value of g(x) is 

A 4.0 

B 4.5 

C 1.5 

D None of the above 

Answer: D 

Explanation: 

Smallest possible value would be at 5-x = x+2 i.e. x= 1.5 as shown 

Substituting we get the smallest value as 3.5. 

Q. 64 The function f(x) = |x – 2| + |2.5 – x| + |3.6 – x|, where x is a real number, attains a minimum at 

A x = 2.3 

B x = 2.5 

C x = 2.7 

D None of the above 

Answer: B 

Explanation: 

f(x) = |x – 2| + |2.5 – x| + |3.6 – x| 

For x belonging to (-infinity to 2), f(x) = 2-x + 2.5-x + 3.6-x = 8.1-3x 

This attains the minimum value at x=2. Value = 2.1 

For x belonging to (2 to 2.5), f(x) = x-2 + 2.5-x + 3.6-x = 4.1-x 

Attains the minimum value at x = 2.5. Value = 1.6 

For x belonging to (2.5 to 3.6), f(x) = x-2 + x-2.5 + 3.6-x = x-0.9 

Attains the minimum at x=2.5, value = 1.6 

For x > 3.6, f(x) = x-2+x-2.5+x-3.6 = 3x – 8.1 

Attains the minimum at x= 3.6, value = 2.7 

So, min value of the function is 1.6 at x=2.5 

Q. 65 How many even integers n, where 100 ≤ n ≤ 200, are divisible neither by seven nor by nine? 

A 40 

B 37 

C 39 

D 38 

Answer: C 

Explanation: 

Between 100 and 200 both included there are 51 even nos. There are 7 even nos which are divisible by 7 and 6 nos which are divisible by 9 and 1 no divisible by both. hence in total 51 – (7+6-1) = 39 

There is one more method through which we can find the answer. Since we have to find even numbers, consider the numbers which are divisible by 14, 18 and 126 between 100 and 200. These are 7, 6 and 1 respectively. 

Q. 66 A positive whole number M less than 100 is represented in base 2 notation, base 3 notation, and base 5 notation. It is found that in all three cases the last digit is 1, while in exactly two out of the three cases the leading digit is 1. Then M equals 

A 31 

B 63 

C 75 

D 91 

Answer: D 

Explanation: 

Since in all three cases the last digit is 1, the number should give remainder 1 when divided individually by 2,3,5 . So the no. may be 31 or 91 . Now 31 in base 2,3 and 5 give the first digit as 1 in all the 3 cases while 91 gives exactly two out of the three cases the leading digit as 1. Hence option D. 

Q. 67 In a 4000 meter race around a circular stadium having a circumference of 1000 meters, the fastest runner and the slowest runner reach the same point at the end of the 5th minute, for the first time after the start of the race. All the runners have the same starting point and each runner maintains a uniform speed throughout the race. If the fastest runner runs at twice the speed of the slowest runner, what is the time taken by the fastest runner to finish the race? 

A 20 min 

B 15 min 

C 10 min 

D 5 min 

Answer: C 

Explanation: 

Let A , B and f,s be the distance traveled and speed of the fastest and the slowest person respectively. Also f=2s so in the given time A=2B. Since the ration of the speeds is 2:1, they will meet at 2-1 points = 1 point. 

Both meet each other for the first time at the starting point . let b travel distance equal to 1 circumference i.e. 1000m so A=2000m . Both meet after 5 min so the slowest is 1000/5=200m/min . So the speed of the fastest is 400m/min. So time taken by A to complete race 4000/400 = 10 min 

Q. 68 Is a44 < b11, given that a = 2 and b is an integer? 

A. b is even 

B. b is greater than 16 

A The questions can be answered by one of the statements, but not by the other 

B The Q. can be answered using either of the two statements alone. 

C The questions can be answered using both the statements together, but cannot be answered using either statement alone. 

D The Q. cannot be answered even by using both the statements together. 

Answer: A 

Explanation: 

If b = 4, then the answer to the Q. is no. If b = 32, then the answer is yes. So, using statement A alone, we cannot answer the question. 

Using statement B alone, we can conclude that a44 < b11 

Q. 69 What are the unique values of b and c in the equation 4x2 + bx + c = 0 if one of the roots of the equation is (- 1/2)? 

A. The second root is 1/2. 

B. The ratio of c and b is 1. 

A The questions can be answered by one of the statements, but not by the other 

B The Q. can be answered using either of the two statements alone. 

C The questions can be answered using both the statements together, but cannot be answered using either statement alone. 

D The Q. cannot be answered even by using both the statements together. 

Answer: B 

Explanation: 

Using statement A, sum of roots = 0 and product of roots = 0, so b = c = 0. 

Using statement B, sum of roots = x – 1/2 = -b/4 and product of roots = c/4 = b/4 = -x/2 

So, we can calculate the values of b and c using either statement alone 

Q. 70 AB is a chord of a circle. AB = 5 cm. A tangent parallel to AB touches the minor arc AB at E. What is the radius of the circle? 

A. AB is not a diameter of the circle. 

B. The distance between AB and the tangent at E is 5 cm. 

A The questions can be answered by one of the statements, but not by the other 

B The Q. can be answered using either of the two statements alone. 

C The questions can be answered using both the statements together, but cannot be answered using either statement alone. 

D The Q. cannot be answered even by using both the statements together. 

Answer: A 

Explanation: 

Let the radius be r. Using statement B alone, r, r-5 and 2.5 form a right-angled triangle. So, we can answer the Q. using statement B alone. 

Q. 71 (1/a2 + 1/a4 + 1/a6 + …) > (1/a + 1/a3 + 1/a5 + …) Is ? 

 

A. 0 < a ≤ 1 

B. One of the roots of the equation 4x2 − 4x + 1 = 0 is a 

A The questions can be answered by one of the statements, but not by the other 

B The Q. can be answered using either of the two statements alone. 

C The questions can be answered using both the statements together, but cannot be answered using either statement alone. 

D The Q. cannot be answered even by using both the statements together. 

Answer: A 

Explanation: 

Consider the first statement: 

When the common ratio is less than 1 we can apply the formula of sum of infinite terms. 

So, LHS = 1/(a2 − 1) 

RHS = a/(a2 − 1) 

If a<1 then LHS<RHS 

If a = 1,then LHS = RHS 

So, we cannot answer the Q. using statement 1 alone 

Using statement 2 alone, we know that a = 1/2. So, RHS > LHS 

Hence, option a) 

Q. 72 D, E, F are the mid points of the sides AB, BC and CA of triangle ABC respectively. What is the area of DEF in square centimeters? 

A. AD = 1 cm, DF = 1 cm and perimeter of DEF = 3 cm 

B. Perimeter of ABC = 6 cm, AB = 2 cm, and AC = 2 cm. 

A The questions can be answered by one of the statements, but not by the other 

B The Q. can be answered using either of the two statements alone. 

C The questions can be answered using both the statements together, but cannot be answered using either statement alone. 

D The Q. cannot be answered even by using both the statements together. 

Answer: B 

Explanation: 

From statement 1 alone, we can infer that the triangle ABC is an equilateral triangle with side = 2 cm Similarly, from statement 2 alone, we can infer that the triangle ABC is an equilateral of side 2 cm So, the Q. can be answered using either statement alone 

Q. 73 At the end of year 1998, Shepard bought nine dozen goats. Henceforth, every year he added p% of the goats at the beginning of the year and sold q% of the goats at the end of the year where p > 0 and q > 0. If Shepard had nine dozen goats at the end of year 2002, after making the sales for that year, which of the following is true? 

A p = q 

B p < q 

C p > q 

D p = q/2 

Answer: C 

Explanation: 

By the end of the year 2002, Shepard bought 4 times and sold 4 times. He is left with the initial number of goats that he had in 1998. If the percentage of goats bought is equal to or lesser than the percentage of goats sold, then there would be a net decrease in the total number of goats. For the number of goats to remain the same, p has to be greater than q, because p% is being calculated in a lesser number and q% is being calculated on a greater number. Hence, p > q. 

Q. 74 Each side of a given polygon is parallel to either the X or the Y axis. A corner of such a polygon is said to be convex if the internal angle is 90° or concave if the internal angle is 270°. If the number of convex corners in such a polygon is 25, the number of concave corners must be 

A 20 

B 0 

C 21 

D 22 

Answer: C 

Explanation: 

Let the total number of sides be x. 

Sum of internal angles in a polygon = (x-2)*180 where x is the number of sides. 

It is given that the polygon has 25 convex sides, then the number of concave sides = x-25 (25*90)+(x-25)*270 = (x-2)180 

x = 46 

Number concave corners = x-25 = 46-25 = 21 

Q. 75 The 288th term of the series a,b,b,c,c,c,d,d,d,d,e,e,e,e,e,f,f,f,f,f,f… is 

A u 

B v 

C w 

D x 

Answer: D 

Explanation: 

1, 2, 3, 4,….n such that the sum is greater than 288 

If n = 24, n(n+1)/2 = 12*25 = 300 

So, n = 24, i.e. the 24th letter in the alphabet is the letter at position 288 in the series 

So, answer = x 

Q. 76 Let p and q be the roots of the quadratic equation x2 − (α − 2)x α − 1 = 0 . What is the minimum possible p2 + q2 

value of ? 

A 0 

B 3 

C 4 

D 5 

Answer: D 

Explanation: 

Let α be equal to k. 

=> f(x) = x2 − (k − 2)x − (k + 1) = 0

p and q are the roots 

=> p+q = k-2 and pq = -1-k 

We know that (p + q)2 = p2 + q2 + 2pq 

=>(k − 2)2 = p2 + q2 + 2(−1 − k

 =>p2 + q2 = k2 + 4 − 4k + 2 + 2

 =>p2 + q2 = k2 − 2k + 6 

This is in the form of a quadratic equation. 

The coefficient of k2 is positive. Therefore this equation has a minimum value. 

We know that the minimum value occurs at x = −b/2

Here a = 1, b = -2 and c = 6 

=> Minimum value occurs at k =2/2  = 1 

If we substitute k = 1 in k2 − 2k + 6 , we get 1 -2 + 6 = 5. 

Hence 5 is the minimum value that p2 + q2  can attain. 

Q. 77 There are two concentric circles such that the area of the outer circle is four times the area of the inner circle. Let A, B and C be three distinct points on the perimeter of the outer circle such that AB and AC are tangents to the inner circle. If the area of the outer circle is 12 square centimeters then the area (in square centimeters) of the triangle ABC would be 

A π√12

B 9/π

C 93/π

D 63/π

Answer: C 

Explanation: 

Let R ,r be radius of big and small circles respectively. We know that R=2r. And since area = 12 ; 

R2 = 12/π

By pythagoras theorem in the small triangle with side ‘x’ we have x = 3 /√π. This triangle is a 30-60-90 right triangle. => Triangle ABC is an equilateral triangle. 

So side of triangle ABC = 2x = 6 /√π

Also Angle OAB = 30 (as the side opposite to the angle is half of the hypotenuse ). 

Hence Angle CAB = 60. 

Hence the required triangle is an equilateral triangle. So area =√3 * (2x)2 / 4 . Substituting we get . 

Area = 9√3/π

Q. 78 Let a, b, c, d be four integers such that a+b+c+d = 4m+1 where m is a positive integer. Given m, which one of the following is necessarily true? 

A The minimum possible value of is a2 + b2 + c2 + d2 4m2 − 2m + 1

B The minimum possible value of is a2 + b2 + c2 + d2 4m2 + 2m + 1

C The maximum possible value of is a2 + b2 + c2 + d2 4m2 − 2m + 1

D The maximum possible value of is a2 + b2 + c2 + d2 4m2 + 2m + 1

Answer: B 

Explanation: 

Taking lowest possible positive value of m i.e. 1 . Such that a+b+c+d=5 , so at least one of them must be greater than 1 , take a=b=c=1 and d=2 

we get a2 + b2 + c2 + d2 = 7 which is equal to 4m2 + 2m + 1 for other values it is greater than 4m2 + 2m + 1 . so option B 

Q. 79 Three horses are grazing within a semi-circular field. In the diagram given below, AB is the diameter of the semi-circular field with center at O. Horses are tied up at P, R and S such that PO and RO are the radii of semi-circles with centers at P and R respectively, and S is the center of the circle touching the two semi circles with diameters AO and OB. The horses tied at P and R can graze within the respective semi-circles and the horse tied at S can graze within the circle centred at S. The percentage of the area of the semicircle with diameter AB that cannot be grazed by the horses is nearest to 

A 20 

B 28 

C 36 

D 40 

Answer: B 

Explanation: 

Let R be radius of big circle and r be radius of circle with centre S. Radius of 2 semicircles is R/2. 

From Right angled triangle OPS, using pythagoras theorem we get 

(r + 0.5R)2 = (0.5R)2 + (R r)2 . We get R=3r . 

Now the area of big semicircle that cannot be grazed is Area of big S.C – area of 2 semicircle – area of small circle = π R2 /2 – 2*π* (0.5R)2/2- πr

= π R2 /2 – 2* π ∗(0.5R)2 π /2 – π ∗(R/3)2

= π R2 /2 – π R2/4 – π ∗(R)2/9

= 5*π * R2/36. this is about 28 % of the area π R2/2 . Hence option B. 

Q. 80 In the figure below, ABCDEF is a regular hexagon and ∠AOF= 90° . FO is parallel to ED. What is the ratio of the area of the triangle AOF to that of the hexagon ABCDEF? 

A 1/12 

B 1/6 

C 1/24 

D 1/18 

Answer: A 

Explanation: 

When the hexagon is divided into a number of similar triangles AOF we get 12 such triangles . Hence the required ratio of area is 1/12

Q. 81 How many three digit positive integers, with digits x, y and z in the hundred’s, ten’s and unit’s place 

respectively, exist such that x < y, z < y and x ≠ 0 ? 

A 245 

B 285 

C 240 

D 320 

Answer: C 

Explanation: 

x, y and z in the hundred’s, ten’s and unit’s place. So y should start from 2 

If y=2 , possible values of x=1 and z = 0,1 .So 2 cases 120,121. 

Also if y=3 , possible values of x=1,2 and z=0,1,2. 

Here 6 three digit nos. possible . 

Similarly for next cases would be 3*4=12,4*5=20,5*6=30,…..,8*9=72 . Adding all we get 240 cases.

Q. 82 A vertical tower OP stands at the center O of a square ABCD. Let h and b denote the length OP and AB respectively. Suppose ∠APB = 60° then the relationship between h and b can be expressed as 

A 2b2 = h2

B 2h2 = b2

C 3b2 = 2h2

D 3h2 = 2b2

Answer: B 

Explanation: 

Consider the triangle APB. ∠P = 600 and AP = BP => APB is an equilateral triangle.

Hence AP = b…(1) 

AC2  = AB2 + BC2 

AC2  = b2 + b2 => AC = √2 × b => AO = AC/2 = b/√2

AP2 = AO2  +OP2 

b2 =b2 /2 + h2  …From (1)

2h2 = b2 

Hence, option B is the correct answer. 

Q. 83 In the triangle ABC, AB = 6, BC = 8 and AC = 10. A perpendicular dropped from B, meets the side AC at D. A circle of radius BD (with center B) is drawn. If the circle cuts AB and BC at P and Q respectively, the AP:QC is equal to 

A 1:1 

B 3:2 

C 4:1 

D 3:8 

Answer: D 

Explanation: 

Let BD = x .Semi-perimeter of triangle ABC = 12. Now by herons formula area of ABC is 24. Also Area = 0.5*x*10 . We get x = 24/5 . AP = 6/5 and CQ = 16/5 . Hence the required ratio is 3:8. 

Q. 84 In the diagram given below, ∠ABD = ∠CDB = ∠PQD = 90° . If AB:CD = 3:1, the ratio of CD: PQ is

A 1 : 0.69 

B 1 : 0.75 

C 1 : 0.72 

D None of the above. 

Answer: B 

Explanation: 

Let BQ = z , QD = y , PQ = x. 

From similar triangles PQD and ABD we have 

(y/x) = (z+y)/3 . 

Also from similar triangles PQB and CBD we have 

(z/x) = z+y . 

Solving we get z = 3*y. 

Now the required ratio is (z+y)/z. 

We get equal to 4/3 which is equal to 1:0.75. 

Q. 85 There are 8436 steel balls, each with a radius of 1 centimeter, stacked in a pile, with 1 ball on top, 3 balls in the second layer, 6 in the third layer, 10 in the fourth, and so on. The number of horizontal layers in the pile is 

A 34 

B 38 

C 36 

D 32 

Answer: C 

Explanation: 

For the given problem , 

n(n + 1)/2 = 8436  which is 

 

n2/2 + ∑n/2 = 8436 which is equal to 

 

n*(n+1)(2n+1)/12 + n*(n+1)/4 = 8436 , solving we get n=36. 

Solving the equation might be lengthy. you can substitute the values in the options to arrive at the answer. 

Q. 86 If the product of n positive real numbers is unity, then their sum is necessarily 

A a multiple of n 

B equal to n + 1/n 

C never less than n 

D a positive integer 

Answer: C 

Explanation: 

 

Let a1, a2….an.the numbers be 

Since the numbers are positive, 

AM GM 

(a1+a2+a3….+an )/ n ≥(a1 ∗ a2…. ∗ an)1/n 

a1 + a2 + a3…. + an

Q. 87 If log32, log3(2x − 5), log3(2x − 7/2) are in arithmetic progression, then the value of x is equal to 

A 5 

B 4 

C 2 

D 3 

Answer: D 

Explanation: 

2log(2x − 5) = log2 + log(2x − 7/2) 

Let 2x = t

=> (t − 5)2 = 2(t − 7/2)

=> t2 + 25 − 10t = 2t − 7 

=> t2 − 12t + 32 = 0

=> t = 8, 4 

Therefore, x = 2 or 3, but 2x > 5, so x = 3  

Q. 88 In the figure below, AB is the chord of a circle with center O. AB is extended to C such that BC = OB. The straight line CO is produced to meet the circle at D. If ∠ACD= y degrees and ∠AOD= x degrees such that x = ky, then the value of k is 

A 3 

B 2 

C 1 

D None of the above. 

Answer: A 

Explanation: 

Since Angle BOC = Angle BCO = y. 

Angle OBC = 180-2y . 

Hence Angle ABO = z = 2y = Angle OAB. 

Now since x is the exterior angle of triangle AOC . 

We have x = z + y = 3y. 

Hence option A. 

Q. 89 In the figure below, the rectangle at the corner measures 10 cm × 20 cm. The corner A of the rectangle is also a point on the circumference of the circle. What is the radius of the circle in cm? 

A 10 cm 

B 40 cm 

C 50 cm 

D None of the above. 

Answer: C 

Explanation: 

As seen in the fig. we have a right angled triangle with sides r ,r-10 , r-20. 

r2 = (r − 10)2 + (r − 20)2 

Using pythagoras we have . 

Solving the equation, we get r = 10 or 50. 

But 10 is not possible , so r = 50. 

Hence the radius is 50. 

Q. 90 Given that −1 ≤ v ≤ 1, −2 ≤ u ≤ −0.5 and −2 ≤ z ≤ −0.5 and w = vz/u , then which of the following is necessarily true? 

A −0.5 ≤ w ≤ 2

B −4 ≤ w ≤ 4

C −4 ≤ w ≤ 2

D −2 ≤ w ≤ −0.5

Answer: B 

Explanation: 

We know w = vz/u so taking max value of u and min value of v and z to get min value of w which is -4. Similarly taking min value of u and max value of v and z to get max value of w which is 4 

Take v = 1, z = -2 and u = -0.5, we get w = 4 

Take v = -1, z = -2 and u = -0.5, we get w = -4 

Q. 91 There are 6 boxes numbered 1,2,… 6. Each box is to be filled up either with a red or a green ball in such a way that at least 1 box contains a green ball and the boxes containing green balls are consecutively numbered. The total number of ways in which this can be done is 

A 5 

B 21 

C 33 

D 60 

Answer: B 

Explanation: 

If there is only 1 green ball, it can be done in 6 ways 

If there are 2 green balls, it can be done in 5 ways. 

If there are 6 green balls, it can be done in 1 way. 

So, the total number of possibilities is 6*7/2 = 21 

Q. 92 Consider the following two curves in the x-y plane: 

y = x3 + x2 + 5 

y = x2 + x + 5 

Which of the following statements is true for −2 ≤ x ≤ 2 ? 

A The two curves intersect once. 

B The two curves intersect twice. 

C The two curves do not intersect 

D The two curves intersect thrice. 

Answer: D 

Explanation: 

Equate the 2 equations we get the value of x = 1 and -1 . Also we notice that there is an intersection at x=0 . hence D 

Q. 93 In a certain examination paper, there are no questions. For j = 1,2 …n, there are 2nj  students who answered j or more questions wrongly. If the total number of wrong answers is 4095, then the value of n is 

A 12 

B 11 

C 10 

D 9 

Answer: A 

Explanation: 

Let there only be 2 questions. 

Thus there are 22−1= 2 students who have done 1 or more questions wrongly, 22−2 = 1 students who have done all 2 questions wrongly . 

Thus total number of wrong answers = 2 + 1 = 3= 2n − 1. 

Now let there be 3 questions. Then j = 1,2,3 

Number of students answering 1 or more questions incorrectly = 4 

Number of students answering 2 or more questions incorrectly = 2 

Number of students answering 3 or more questions incorrectly = 1 

Total number of incorrect answers = 1(3)+(2-1)*2+(4-2)*1 = 7 = 23 − 1

According to the Q. , the total number of wrong answers = 4095 = 212 − 1 .

Hence Option A.  

Q. 94 If x, y, z are distinct positive real numbers the (x2(y + z) + y2(x + z) + z2(x + y))/xyz would be 

A greater than 4 

B greater than 5 

C greater than 6 

D None of the above 

Answer: C 

Explanation: 

For the given expression value of x,y,z are distinct positive integers . So the value of expression will always be greater than value when all the 3 variables are equal . substitute x=y=z we get minimum value of 6 . 

(x2(y + z) + y2(x + z) + z2(x + y))/xyz = x/z + x/y + y/z + y/x + z/y + z/x 

Applying AM greater than or equal to GM, we get minimum sum = 6 

Q. 95 A graph may be defined as a set of points connected by lines called edges. Every edge connects a pair of points. Thus, a triangle is a graph with 3 edges and 3 points. The degree of a point is the number of edges connected to it. For example, a triangle is a graph with three points of degree 2 each. Consider a graph with 12 points. It is possible to reach any point from any point through a sequence of edges. The number of edges, e, in the graph must satisfy the condition 

A 11 ≤ e ≤ 66

B 10 ≤ e ≤ 66

C 11 ≤ e ≤ 65

D 0 ≤ e ≤ 11

Answer: A 

Explanation: 

Take any 12 points. 

The maximum number of edges which can be drawn through these 12 points are 12C2 = 66 

The minimum number of edges which can be drawn through these 12 points are 12-1 = 11 as the resulting figure need not be closed. It might be open. 

Q. 96 The number of positive integers n in the range 12 ≤ n ≤ 40 such that the product (n -1)*(n – 2)*…*3*2*1 is not divisible by n is 

A 5 

B 7 

C 13 

D 14 

Answer: B 

Explanation: 

positive integers n in the range 12 ≤ n ≤ 40 such that the product (n -1)*(n – 2)*…*3*2*1 is not divisible by n, implies that n should be a prime no. So there are 7 prime nos. in the given range. Hence option B.  

Q. 97 Let T be the set of integers {3,11,19,27,…451,459,467} and S be a subset of T such that the sum of no two elements of S is 470. The maximum possible number of elements in S is 

A 32 

B 28 

C 29 

D 30 

Answer: D 

Explanation: 

No. of terms in series T , 3+(n-1)*8 = 467 i.e. n=59. 

Now S will have at least 59 terms i.e 29 . 

Also the sum of 29th term and 30th term is less than 470. 

Hence, the maximum possible elements in S is 30. 

 

Instructions 

DIRECTIONS for the following two questions: Answer the questions on the basis of the information given below. 

Some children were taking free throws at the basketball court in school during lunch break. Below are some facts about how many baskets these children shot. 

i. Ganesh shot 8 baskets less than Ashish. 

ii. Dhanraj and Ramesh together shot 37 baskets. 

iii. Jugraj shot 8 baskets more than Dhanraj. 

iv. Ashish shot 5 baskets more than Dhanraj. 

v. Ashish and Ganesh together shot 40 baskets. 

 

Q. 98 Which of the following statements is true? 

A Ramesh shot 18 baskets and Dhanraj shot 19 baskets 

B Ganesh shot 24 baskets and Ashish shot 16 baskets 

C Jugraj shot 19 baskets and Dhanraj shot 27 baskets 

D Dhanraj shot 11 baskets and Ashish shot 16 baskets 

Answer: A 

Explanation: 

Let a,d,j,g be the shots put by Ashish, Dhanraj, Ganesh and Jugraj respectively. 

According to given conditions we have , 

g = a-8; d + r = 37; j = d + 8; a = 5 + d; a + g = 40 

Solving these equations, we have a = 24, d = 19 and j = 27 and r = 18. Hence option A is the correct answer. 

 

Instructions 

DIRECTIONS for the following three questions: Answer the questions on the basis of the information given below. 

A city has two perfectly circular and concentric ring roads, the outer ring road (OR) being twice as long as the inner ring road (IR). There are also four (straight line) chord roads from E1, the east end point of OR to N2, the north end point of IR; from N1, the north end point of OR to W2, the west end point of IR; from W1, the west end point of OR, to S2, the south end point of IR; and from S1 the south end point of OR to E2, the east end point of IR. Traffic moves at a constant speed of 30π km/hr on the OR road, 20π km/hr on the IR road, and 15√5 km/hr on all the chord roads. 

Q. 99 Amit wants to reach N2 from S1. It would take him 90 minutes if he goes on minor arc S1 – E1 on OR, and then on the chord road E1 – N2. What is the radius of the outer ring road in kms? 

A 60 

B 40 

C 30 

D 20 

Answer: C 

Explanation: 

We know that the total time taken is 1.5 hrs. Calculating the individual time taken and the adding and then equating to 1.5.

R)/(2∗30∗π) + (√5∗R)/ 2∗15∗√5 = 1.5 

solving we get R=30. 

Q. 100 Amit wants to reach E2 from N1 using first the chord N1 – W2 and then the inner ring road. What will be his travel time in minutes on the basis of information given in the above question? 

A 60 

B 45. 

C 90 

D 105 

Answer: D 

Explanation: 

Let the radii of 2 circles be R and r respectively such that R=2*r. Triangle O $$N_2E_1 And all the other 3 similar triangles form a right angle at the centre. So using pythagoras theorem the value of chords come out to be 52∗R/2.Hence the total distance traveled is 52* R/2 + 0.5*R*pi. Total time required can be calculated by distance / speed which comes out to be 3.5*R. Among options only 105 is integral multiple of 3.5. 

 

 

Verbal 

Instructions 

At the heart of the enormous boom in wine consumption that has taken place in the English speaking world over the last two decades or so is a fascinating, happy paradox. In the days when wine was exclusively the preserve of a narrow cultural elite, bought either at auctions or from gentleman wine merchants in wing collars and bow-ties, to be stored in rambling cellars and decanted to order by one’s butler, the ordinary drinker didn’t get a look-in. Wine was considered a highly technical subject, in which anybody without the necessary ability could only fall flat on his or her face in embarrassment. It wasn’t just that you needed a refined aesthetic sensibility for the stuff if it wasn’t to be hopelessly wasted on you. It required an intimate knowledge of what came from where, and what it was supposed to taste like. 

Those were times, however, when wine appreciation essentially meant a familiarity with the great French classics, with perhaps a smattering of other wines — like sherry and port. That was what the wine trade dealt in. These days, wine is bought daily in supermarkets and high-street chains to be consumed that evening, hardly anybody has a cellar to store it in and most don’t even possess a decanter. Above all, the wines of literally dozens of countries are available on our market. When a supermarket offers its customers a couple of fruity little numbers from Brazil, we scarcely raise an eyebrow. 

It seems, in other words, that the commercial jungle that wine has now become has not in the slightest deterred people from plunging adventurously into the thickets in order to taste and see. Consumers are no longer intimidated by the thought of needing to know their Pouilly-Fume from their Pouilly-Fuisse, just at the very moment when there is more to know than ever before. 

The reason for this new mood of confidence is not hard to find. It is on every wine label from Australia, New Zealand, South Africa and the United States: the name of the grape from which the wine is made. At one time that might have sounded like a fairly technical approach in itself. Why should native English-speakers know what Cabernet Sauvignon or Chardonnay were? The answer lies in the popularity that wines made from those grape varieties now enjoy. Consumers effectively recognize them as brand names, and have acquired a basic lexicon of wine that can serve them even when confronted with those Brazilian upstarts. 

In the wine heartlands of France, they are scared to death of that trend—not because they think their wine isn’t as good as the best from California or South Australia (what French winemaker will ever admit that?) but because they don’t traditionally call their wines Cabernet Sauvignon or Chardonnay. They call them Château Ducru Beaucaillou or Corton Charlemagne, and they aren’t about the change. Some areas, in the middle of southern France, have now produced a generation of growers using the varietal names on their labels and are tempting consumers back to French wine. It will be an uphill struggle, but there is probably no other way if France is to avoid simply becoming a specialty source of old fashioned wines for old fashioned connoisseurs. 

Wine consumption was also given a significant boost in the early 1990s by the work of Dr. Serge Renaud, who has spent many years investigating the reasons for the uncannily low incidence of coronary heart disease in the south of France. One of his major findings is that the fat-derived cholesterol that builds up in the arteries and can eventually lead to heart trouble, can be dispersed by the tannins in wine. Tannin is derived from the skins of grapes, and is therefore present in higher levels in red wines, because they have to be infused with their skins to attain the red colour. That news caused a 

huge upsurge in red wine consumption in the United States. It has not been accorded the prominence it deserves in the UK, largely because the medical profession still sees all alcohol as a menace to health, and is constantly calling for it to be made prohibitively expensive. Certainly, the manufacturers of anticoagulant drugs might have something to lose if we all got the message that we would do just as well by our hearts by taking half a bottle of red wine every day! 

Q. 101 The tone that the author uses while asking “what French winemaker will ever admit that?” is best described as 

A caustic 

B satirical 

C critical 

D hypocritical 

Answer: B 

Explanation: 

The tone which the author uses while asking “what French winemaker will ever admit that? ” is not at all harsh , so option a) is out . Also the author doesn’t criticize while asking the Q. ,so Option c ) is clearly not the answer. We don’t find any author professing any feelings which he doesn’t have , hence the tone is not hypocritical. Option b ,suits the best i.e. his tone is satirical. 

Q. 102 What according to the author should the French do to avoid becoming a producer of merely old fashioned wines? 

A Follow the labeling strategy of the English-speaking countries 

B Give their wines English names 

C Introduce fruity wines as Brazil has done 

D Produce the wines that have become popular in the English-speaking world 

Answer: A 

Explanation: 

Refer to the part of the passage which says ‘ …not because they think their wine isn’t as good as the best from California …and they aren’t about the change…probably no other way if France is to avoid simply becoming a specialty source of old fashioned wines for old fashioned connoisseurs. ‘ . 

From this, we can see that the author feels that the French should adopt the labeling strategy of the English-speaking countries to avoid becoming a producer of merely old-fashioned wines. 

Option a) is the correct answer. 

Q. 103 The development which has created fear among winemakers in the wine heartland of France is the 

A tendency not to name wines after the grape varieties that are used in the wines 

B ‘education’ that consumers have derived from wine labels from English speaking countries. C new generation of local winegrowers who use labels that show names of grape varieties 

D ability of consumers to understand a wine’s qualities when confronted with “Brazilian upstarts”. 

Answer: B 

Explanation: 

Consider the following part of the passage ‘ .. Consumer effectively recognize them as brand names, and have acquired a basic lexicon of wine …. with those Brazilian upstarts. In the wine heartlands of France, they are scared to death of that trend.. ‘. So, above all, French winemakers fear the knowledge or education that the consumers have derived from wine labels from the English speaking countries. Option b) is the correct answer. 

Q. 104 Which one of the following, if true, would provide most support for Dr. Renaud’s findings about the effect of tannins? 

A A survey showed that film celebrities based in France have a low incidence of coronary heart disease. 

B Measurements carries out in southern France showed red wine drinkers had significantly higher levels of coronary heart incidence than white wine drinkers did. 

C Data showed a positive association between sales of red wine and incidence of coronary heart disease. 

D Long-term surveys in southern France showed that the incidence of coronary heart disease was significantly lower in red wine drinkers than in those who did not drink red wine. 

Answer: D 

Explanation: 

Dr. Renaud findings suggest that fat-derived cholesterol can be dispersed by the tannins in wine. So, a survey that validates this finding would provide the most support. The survey in option d) is precisely one such survey. Option d) is the correct answer. 

Q. 105 Which one of the following CANNOT be reasonably attributed to the labeling strategy of followed by wine producers in English speaking countries? 

A Consumers buy wines on the basis of their familiarity with a grape variety’s name. 

B Even ordinary customers now have more access to technical knowledge about wine. 

C Consumers are able to appreciate better quality wines. 

D Some non-English speaking countries like Brazil indicate grape variety names on their labels. Answer: C 

Explanation: 

Consumers’ appreciation of better quality wines is something that does not come because of the labeling or branding. So, this appreciation cannot be attributed to the labeling strategy followed by wine producers in English speaking countries. 

Option c) is the correct answer. 

 

Instructions 

Right through history, imperial powers have clung to their possessions to death. Why, then, did Britain in 1947 give up the jewel in its crown, India? For many reasons. The independence struggle exposed the hollowness of the white man’s burden. Provincial self-rule since 1935 paved the way for full self-rule. Churchill resisted independence, but the Labour government 

of Atlee was anti-imperialist by ideology. Finally, the Royal Indian Navy mutiny in 1946 raised fears of a second Sepoy mutiny, and convinced British waverers that it was safer to withdraw gracefully. But politico-military explanations are not enough. The basis of empire was always money. The end of empire had much to do with the fact that British imperialism had ceased to be profitable. World War II left Britain victorious but deeply indebted, needing Marshall Aid and loans from the World Bank. This constituted a strong financial case for ending the no-longer profitable empire. 

Empire building is expensive. The US is spending one billion dollars a day in operations in Iraq that fall well short of full scale imperialism. Through the centuries, empire building was costly, yet constantly undertaken because it promised high returns. The investment was in armies and conquest. The returns came through plunder and taxes from the conquered. 

No immorality was attached to imperial loot and plunder. The biggest conquerors were typically revered (hence titles like Alexander the Great, Akbar the Great, and Peter the Great). The bigger and richer the empire, the more the plunderer was admired. This mindset gradually changed with the rise of new ideas about equality and governing for the public good, ideas that culminated in the French and American revolutions. 

Robert Clive was impeached for making a little money on the side, and so was Warren Hastings. The white man’s burden came up as a new moral rationale for conquest. It was supposedly for the good of the conquered. This led to much muddled hypocrisy. On the one hand, the empire needed to be profitable. On the other hand, the white man’s burden made brazen loot impossible. 

An additional factor deterring loot was the 1857 Sepoy Mutiny. Though crushed, it reminded the British vividly that they were a tiny ethnic group who could not rule a gigantic subcontinent without the support of important locals. After 1857, the British stopped annexing one princely state after another, and instead treated the princes as allies. Land revenue was fixed in absolute terms, partly to prevent local unrest and partly to promote the notion of the white man’s burden. The empire proclaimed itself to be a protector of the Indian peasant against exploitation by Indian elites. This was denounced as hypocrisy by nationalists like Dadabhoy Naoroji in the 19th century, who complained that land taxes led to an enormous drain from India to Britain. Objective calculations by historians like Angus Maddison suggest a drain of perhaps 1.6 percent of Indian Gross National Product in the 19th century. But land revenue was more or less fixed by the Raj in absolute terms, and so its real value diminished rapidly with inflation in the 20th century. By World War II, India had ceased to be a profit center for the British Empire. 

Historically, conquered nations paid taxes to finance fresh wars of the conqueror. India itself was asked to pay a large sum at the end of World War I to help repair Britain’s finances. But, as shown by historian Indivar Kamtekar, the independence movement led by Gandhiji changed the political landscape, and made mass taxation of India increasingly difficult. By World War II, this had become politically impossible. Far from taxing India to pay for World War II, Britain actually began paying India for its contribution of men and goods. Troops from white dominions like Australia; Canada and New Zealand were paid for entirely by these countries, but Indian costs were shared by the British government. Britain paid in the form of nonconvertible sterling balances, which mounted swiftly. The conqueror was paying the conquered, undercutting the profitability on which all empire is founded. Churchill opposed this, and wanted to tax India rather than owe it money. But he was overruled by Indian hands who said India would resist payment, and paralyze the war effort. Leo Amery, Secretary of State for India, said that when you are driving in a taxi to the station to catch a life-or-death train, you do not loudly announce that you have doubts whether to pay the fare. Thus, World War II converted India from a debtor to a creditor with over one billion pounds in sterling balances. Britain, meanwhile, became the biggest debtor in the world. It’s not worth ruling over people you are afraid to tax. 

Q. 106 Why didn’t Britain tax India to finance its World War II efforts? 

A  Australia, Canada and New Zealand had offered to pay for Indian troops. 

B India has already paid a sufficiently large sum during World War I. 

C It was afraid that if India refused to pay, Britain’s war efforts would be jeopardized. 

D The British empire was built on the premise that the conqueror pays the conquered. 

Answer: C 

Explanation: 

Consider the following lines from the passage: “But he was overruled by Indian hands who said India would resist payment and paralyse the war effort”. From this, we can understand the reason why the British didn’t tax India to finance its war efforts. It was afraid that if India refused to pay, Britain’s war efforts would be jeopardized. Option c) is the correct answer. 

Q. 107 What was the main lesson the British learned from the Sepoy Mutiny of 1857. 

A That the local princes were allies, not foes. 

B That the land revenue from India would decline dramatically. 

C That the British were a small ethnic group. 

D That India would be increasingly difficult to rule. 

Answer: C 

Explanation: 

Refer to the 5th paragraph. It says “Though crushed, it reminded the British vividly that they were a tiny ethnic group who could not rule a gigantic subcontinent without the support of the important locals”. From this, we can understand that the main lesson that the British learnt from the Sepoy Mutiny of 1857 was that they were a small ethnic group. Option c) is the correct answer

Q. 108 Which of the following was NOT a reason for the emergence of the ‘white man’s burden’ as a new rationale for empire-building in India? 

A The emergence of the idea of the public good as an element of governance. 

B The decreasing returns from imperial loot and increasing costs of conquest. 

C The weakening of the immorality attached to an emperor’s looting behaviour. 

D A growing awareness of the idea of equality among peoples. 

Answer: B 

Explanation: 

If the returns from conquest decreased and the costs increased, it wouldn’t make sense to continue empire-building in India. So, the sentence in option b) is not a reason for the emergence of the ‘white man’s burden’ as a new rationale for empire-building in India. 

Q. 109 Which of the following best captures the meaning of the ‘white man’s burden’, as it is used by the author? 

 

A The British claim to a civilizing mission directed at ensuring the good of the natives. 

B The inspiration for the French and American revolutions. 

C The resource drain that had to be borne by the home country’s white population. 

D An imperative that made open looting of resources impossible. 

Answer: A 

Explanation: 

White man’s burden refers to the claim made by the British that the natives of the conquered countries were in need of the ‘good’ provided by them. This was a justification for their conquests. Option a) captures this idea succinctly. 

Q. 110 Which one of the following best expresses the main purpose of the author? 

A To present the various reasons that can lead to the collapse of an empire and the granting of independence of the subjects of an empire. 

B To point out the critical role played by the ‘white man’s burden’ in making a colonizing power give up its claims to native possessions. 

C To highlight the contradictory impulse underpinning empire building which is a costly business but very attractive at the same time. 

D To illustrate how erosion of the financial basis of an empire supports the granting of independence to an empire’s constituents. 

Answer: D 

Explanation: 

Throughout the passage, the author talks about the various financial reasons for conquest and explains how the British 

were forced to grant independence when their returns from India diminished after the war. The main idea of the passage is to illustrate how the erosion of the financial basis of an empire supports the granting of independence to an empire’s constituents. Option d) is the correct answer.  

 

Instructions 

The controversy over genetically modified food continues unabated in the West. Genetic modification (GM) is the science by which the genetic material of a plant is altered, perhaps to make it more resistant to pests or killer weeds, or to enhance its nutritional value. Many food biotechnologists claim that GM will be a major contribution of science to mankind in the 21st century. On the other hand, large numbers of opponents, mainly in Europe, claim that the benefits of GM are a myth propagated by multinational corporations to increase their profits, that they pose a health hazard, and have therefore called for government to ban the sale of genetically-modified food. 

The anti-GM campaign has been quite effective in Europe, with several European Union member countries imposing a virtual ban for five years over genetically-modified food imports. Since the genetically-modified food industry is particularly strong in the United States of America, the controversy also constitutes another chapter in the US-Europe skirmishes which have become particularly acerbic after the US invasion of Iraq. 

To a large extent, the GM controversy has been ignored in the Indian media, although Indian biotechnologists have been quite active in GM research. Several groups of Indian biotechnologists have been working on various issues connected with crops grown in India. One concrete achievement which has recently figured in the news is that of a team led by the former vice-chancellor of Jawaharlal Nehru university, Asis Datta — it has successfully added an extra gene to potatoes to enhance the protein content of the tuber by at least 30 percent. It is quite likely that the GM controversy will soon hit the headlines in India since a spokesperson of the Indian Central government has recently announced that the government may use the protato in its midday meal programme for schools as early as next year. 

Why should “scientific progress”, with huge potential benefits to the poor and malnourished, be so controversial? The anti GM lobby contends that pernicious propaganda has vastly exaggerated the benefits of GM and completely evaded the costs which will have to be incurred if the genetically-modified food industry is allowed to grow unchecked. In particular, they allude to different types of costs. 

This group contends that the most important potential cost is that the widespread distribution and growth of genetically modified food will enable the corporate world (alias the multinational corporations – MNCs) to completely capture the food chain. A “small” group of biotech companies will patent the transferred genes as well as the technology associated with them. They will then buy up the competing seed merchants and seed-breeding centers, thereby controlling the production of food at every possible level. Independent farmers, big and small, will be completely wiped out of the food industry. At best, they will be reduced to the status of being subcontractors. 

This line of argument goes on to claim that the control of the food chain will be disastrous for the poor since the MNCs, guided by the profit motive, will only focus on the high-value food items demanded by the affluent. Thus, in the long run, the production of basic staples which constitute the food basket of the poor will taper off. However, this vastly overestimates the power of the MNCs. Even if the research promoted by them does focus on the high-value food items, much of biotechnology research is also funded by governments in both developing and developed countries. Indeed, the protato is a by-product of this type of research. If the protato passes the field trials, there is no reason to believe that it cannot be marketed in the global potato market. And this type of success story can be repeated with other basic food items. 

The second type of cost associated with the genetically modified food industry is environmental damage. The most common type of “genetic engineering” involved gene modification in plants designed to make them resistant to applications of weed-killers. This then enables farmers to use massive dosages of weedkillers so as to destroy or wipe out all competing varieties of plants in their field. However, some weeds through genetically-modified pollen contamination may acquire resistance to a variety of weed-killers. The only way to destroy these weeds is through the use of ever stronger herbicides which are poisonous and linger on in the environment. 

Q. 111 The author doubts the anti-GM lobby’s contention that MNC control of the food chain will be disastrous for the poor because 

A MNCs will focus on high-value food items. 

B MNCs are driven by the motive of profit maximization. 

C MNCs are not the only group of actors in genetically-modified food research. 

D Economic development will help the poor buy MNC-produced food. 

Answer: C 

Explanation: 

By the line “Even if the research promoted by them ………………. in both developing and developed countries”, we can say that not only MNCs but also governments are involved in the research development. 

So, MNCs are not the only group actors that are involved in genetically modified food research. Hence, option C is the answer. 

Q. 112 Using the clues in the passage, which of the following countries would you expect to be in the forefront of the anti-GM campaign? 

A USA and Spain. 

B India and Iraq. 

C Germany and France. 

D Australia and New Zealand. 

Answer: C 

Explanation: 

According to the passage, European nations are anti GM. So, among the given options we must select those countries that are present in Europe. 

USA is not in Europe => option A is wrong. 

India is not in Europe => option B is wrong. 

Australia is not in Europe => option D is wrong. 

Both Germany and France are in Europe => option C is the answer. 

 

Q. 113 Genetic modification makes plants more resistant to killer weeds. However, this can lead to environmental damage by 

A wiping out competing varieties of plants which now fall prey to killer weeds. 

B forcing application of stronger herbicides to kill weeds which have become resistant to weak herbicides. C forcing application of stronger herbicides to keep the competing plants weed-free. 

D not allowing growth of any weeds, thus reducing soil fertility. 

Answer: B 

Explanation: 

Refer to the last lines of the passage:”However, some weeds through genetically-modified pollen contamination may acquire resistance to a variety of weed-killers. The only way to destroy these weeds is through the use of ever-stronger herbicides which are poisonous and linger on in the environment.” This line indicates the point made in 2 that once the weeds acquire resistance to weak herbicides, we have to apply stronger ones to eradicate them. 

Q. 114 According to the passage, biotechnology research 

A is of utility only for high value food items. 

B is funded only by multinational corporations. 

C allows multinational corporations to control the food basket of the poor. 

D addresses the concerns of developed and developing countries. 

Answer: D 

Explanation: 

Refer to the lines made in the paragraph:”Even if the research promoted by them does focus on the high-value food items, much of biotechnology research is also funded by governments in both developing and developed countries. Indeed, the protato is a by-product of this type of research. If the protato passes the field trials, there is no reason to believe that it cannot be marketed in the global potato market. And this type of success story can be repeated with other basic food items.” Here the author wants to illustrate that biotechnology resarch helps to address the concerns of the developing countries. For this illustration, the author gives the exmaple of potatoes. 

Q. 115 Which of the following about the Indian media’s coverage of scientific research does the passage seem to suggest? 

A Indian media generally covers a subject of scientific importance when its mass application is likely. 

B Indian media’s coverage of scientific research is generally dependent on MNCs interests. 

C Indian media, in partnership with the government, is actively involved in publicizing the results of scientific research. 

D Indian media only highlights scientific research which is funded by the government. 

Answer: A 

Explanation: 

Refer to the following lines in the passage:”It is quite likely that the GM controversy will soon hit the headlines in India since a spokesperson of the Indian Central government has recently announced that the government may use the protato in its midday meal programme for schools as early as next year. Why should “scientific progress”, with huge potential benefits to the poor and malnourished, be so controversial?” Here the author wants to highlight that the scientific progress which has a huge impact on large number of people is likely to be covered by the media. 

 

Instructions 

Social life is an outflow and meeting of personality, which means that its end is the meeting of character, temperament, and sensibility, in which our thoughts and feelings, and sense perceptions are brought into play at their lightest and yet keenest. 

This aspect, to my thinking, is realized as much in large parties composed of casual acquaintances or even strangers, as in intimate meetings of old friends. I am not one of those superior persons who hold cocktail parties in contempt, looking upon them as barren or at best as very tryingly kaleidoscopic places for gathering, because of the strangers one has to meet in them; which is no argument, for even our most intimate friends must at one time have been strangers to us. These large gatherings will be only what we make of them if not anything better, they can be as good places to collect new friends from as the slave markets of Istanbul were for beautiful slaves or New Market for race horses. 

But they do offer more immediate enjoyment. For one thing, in them one can see the external expression of social life in appearance and behaviour at its widest and most varied where one can admire beauty of body or air, hear voices remarkable either for sweetness of refinement, look on elegance of clothes or deportment. What is more, these parties are schools for training in sociability, for in them we have to treat strangers as friends. So, in them we see social sympathy in widest commonalty spread, or at least should. We show an atrophy of the natural human instinct of getting pleasure and happiness out of other human beings if we cannot treat strangers as friends for the moment. And I would go further and paraphrase Pater to say that not to be able to discriminate every moment some passionate attitude in those about us, even when we meet them casually, is on this short day of frost and sun which out life is, to sleep before evening. 

So, it will be seen that my conception of social life is modest, for it makes no demands on what we have, though it does make some on what we are. Interest, wonder, sympathy, and love, the first two leading to the last two, are the psychological prerequisites for social life; and the need for the first two must not be underrated. We cannot make the most even of our intimate social life unless we are able to make strangers of our oldest friends everyday by discovering unknown areas in their personality, and transform them into new friends. In sum, social life is a function of vitality. 

It is tragic, however, to observe that it is these very natural springs of social life which are drying up among us. It is becoming more and more difficult to come across fellow-feeling for human beings as such in our society and in all its strata. 

In the poor middle class, in the course of all my life. I have hardly seen any social life properly so-called. Not only has the grinding routine of making a living killed all desire for it in them, it has also generated a standing mood of peevish hostility to other human beings. Increasing economic distress in recent years has infinitely worsened this state of affairs, and has also brought a sinister addition class hatred. This has become the greatest collective emotional enjoyment of the poor middle class, and indeed they feel most social when they form a pack, and snarl or howl at people who are better off than they. 

Their most innocent exhibition of sociability is seen when they spill out from their intolerable homes into the streets and bazaars. I was astonished to see the milling crowds in the poor suburbs of Calcutta. But even there a group of flippant young loafers would put on a conspiratorial look if they saw a man in good clothes passing by them either on foot or in a car. I had borrowed a car from a relative to visit a friend in one of these suburbs, and he became very anxious when I had not returned before dusk. Acid and bombs, he said, were thrown at card almost every evening in that area. I was amazed. But I also know as a fact that my brother was blackmailed to pay five rupees on a trumped up charge when passing in a car through one such locality. 

The situation is differently inhuman, but not a whit more human, among the well-to-do. Kindliness for fellow human beings has been smothered in them, taken as a class, by the arrogance of worldly position, which among the Bengalis who show this snobbery is often only a third-class position. 

Q. 116 The word ‘they’ in the first sentence of the third paragraph refers to 

A Large parties consisting of casual acquaintances and strangers. 

B Intimate meetings of old friends. 

C New friends. 

D Both (1) and (2). 

Answer: A 

Explanation: 

Refer to the last lines of the second paragraph:”These large gatherings will be only what we make of them if not anything better, they can be as good places to collect new friends from as the slavemarkets of Istanbul were for beautiful slaves or New Market for race horses.” 

Here “They” refers to the large gatherings of casual acquaintances as illustrated in these lines

Q. 117 In this passage the author is essentially 

A showing how shallow our social life is. 

B poking fun at the lower middle class people who howl at better off people. 

C lamenting the drying up of our real social life. 

D criticizing the upper class for lavish showy parties. 

Answer: C 

Explanation: 

The passage starts with the author’s perception of social life. In the whole passage the author criticises the fact that we do not possess any social life. The author also gives his opinions of social life and then moves on to lament our little social life we have left. Option b and d are not clearly the main subject of the author. Option a is inappropriate as the author do not highlight our real social life but rather complaints of the social life. 

Q. 118 The author’s conception of ‘social life’ requires that 

A People attend large gatherings. 

B People possess qualities like wonder and interest. 

C People do not spend too much time in the company of intimate friends. 

D Large parties consist of casual acquaintances and intimate friends. 

Answer: B 

Explanation: 

The author’s conception of social life is mentioned in the fourth paragraph where he says wonder and interest. Hence B is the correct answer. 

Q. 119 The word ‘discriminate’ in the last sentence of the third paragraph means 

A recognize. 

B count. 

C distinguish. 

D analyse. 

Answer: A 

Explanation: 

The author is trying to say that people are not able to recognize the passionate attitude. 

So, the word “discriminate” in this context means “recognize”.  

Q. 120 What is the author trying to show through the two incidents in the paragraph beginning, “Their most innocent exhibition of sociability…”? 

A The crowds in poor Calcutta suburbs can turn violent without any provocation. 

B Although poor, the people of poor Calcutta suburbs have a rich social life. 

C It is risky for rich people to move around in poor suburbs. 

D Achieving a high degree of sociability foes not stop the poor from hating the rich. 

Answer: D 

Explanation: 

The author has not mentioned that crowd in poor Calcutta can turn violent => option A is wrong. B negates the statements said by the author in the passage. 

C is too generalized to be the answer for this question. 

Hence, option D is the answer. 

Instructions 

Modern science, exclusive of geometry, is a comparatively recent creation and can be said to have originated with Galileo and Newton. Galileo was the first scientist to recognize clearly that the only way to further our understanding of the physical world was to resort to experiment. However obvious Galileo’s contention may appear in the light of our present knowledge, it remains a fact that the Greeks, in spite of their proficiency in geometry, never seem to have realized the importance of experiment. To a certain extent this may be attributed to the crudeness of their instruments of measurement. Still an excuse of this sort can scarcely be put forward when the elementary nature of Galileo’s experiments 

and observations is recalled. Watching a lamp oscillate in the cathedral of Pisa, dropping bodies from the leaning tower of Pisa, rolling balls down inclined planes, noticing the magnifying effect of water in a spherical glass vase, such was the nature of Galileo’s experiments and observations. As can be seen, they might just as well have been performed by the Greeks. At any rate, it was thanks to such experiments that Galileo discovered the fundamental law of dynamics, according to which the acceleration imparted to a body is proportional to the force acting upon it. 

The next advance was due to Newton, the greatest scientist of all time if account be taken of his joint contributions to mathematics and physics. As a physicist, he was of course an ardent adherent of the empirical method, but his greatest title to fame lies in another direction. Prior to Newton, mathematics, chiefly in the form of geometry, had been studied as a fine art without any view to its physical applications other than in very trivial cases. But with Newton all the resources of mathematics were turned to advantage in the solution of physical problems. Thenceforth mathematics appeared as an instrument of discovery, the most powerful one known to man, multiplying the power of thought just as in the mechanical domain the lever multiplied our physical action. It is this application of mathematics to the solution of physical problems, this combination of two separate fields of investigation, which constitutes the essential characteristic of the Newtonian method. Thus problems of physics were metamorphosed into problems of mathematics. 

But in Newton’s day the mathematical instrument was still in a very backward state of development. In this field again Newton showed the mark of genius by inventing the integral calculus. As a result of this remarkable discovery, problems, which would have baffled Archimedes, were solved with ease. We know that in Newton’s hands this new departure in scientific method led to the discovery of the law of gravitation. But here again the real significance of Newton’s achievement lay not so much in the exact quantitative formulation of the law of attraction, as in his having established the presence of law and order at least in one important realm of nature, namely, in the motions of heavenly bodies. Nature thus exhibited rationality and was not mere blind chaos and uncertainty. To be sure, Newton’s investigations had been concerned with but a small group of natural phenomena, but it appeared unlikely that this mathematical law and order should turn out to be restricted to certain special phenomena; and the feeling was general that all the physical processes of nature would prove to be unfolding themselves according to rigorous mathematical laws. 

When Einstein, in 1905, published his celebrated paper on the electrodynamics of moving bodies, he remarked that the difficulties, which surrouned the equations of electrodynamics, together with the negative experiments of Michelson and others, would be obviated if we extended the validity of the Newtonian principle of the relativity of Galilean motion, which applies solely to mechanical phenomena, so as to include all manner of phenomena: electrodynamics, optical etc. When extended in this way the Newtonian principle of relativity became Einstein’s special principle of relativity. Its significance lay in its assertion that absolute Galilean motion or absolute velocity must ever escape all experimental detection. Henceforth absolute velocity should be conceived of as physically meaningless, not only in the particular ream of mechanics, as in Newton’s day, but in the entire realm of physical phenomena. Einstein’s special principle, by adding increased emphasis to this relativity of velocity, making absolute velocity metaphysically meaningless, created a still more profound distinction between velocity and accelerated or rotational motion. This latter type of motion remained absolute and real as before. It is most important to understand this point and to realize that Einstein’s special principle is merely an extension of the validity of the classical Newtonian principle to all classes of phenomena. 

Q. 121 According to the author, why did the Greeks NOT conduct experiments to understand the physical world? 

A Apparently they did not think it necessary to experiment. 

B They focused exclusively on geometry. 

C Their instruments of measurement were very crude. 

D The Greeks considered the application of geometry to the physical world more important. Answer: A 

Explanation: 

Options B and D negate the information given in the passage => B and D are incorrect. 

C is stated in the passage but does not answer the question. 

Option A is the correct answer. 

Q. 122 The statement “Nature thus exhibited rationality and was not mere blind chaos and uncertainty” suggests that 

A problems that had baffled scientists like Archimedes were not really problems. 

B only a small group of natural phenomena was chaotic. 

C physical phenomena conformed to mathematical laws. 

D natural phenomena were evolving towards a less chaotic future. 

Answer: C 

Explanation: 

In the second line after the line mentioned in the question, the author says that “the feeling was general that all the physical processes of nature would prove to be unfolding themselves according to the rigorous mathematical laws”. 

Option C is the answer. 

Q. 123 Newton may be considered one of the greatest scientists of all time because he 

A discovered the law of gravitation. 

B married physics with mathematics. 

C invented integral calculus. 

D started the use of the empirical method in science. 

Answer: B 

Explanation: 

The author says that “The next advance was due to Newton, the greatest scientist of all time if account be taken of his joint contributions to mathematics and physics.” 

Joint contributions is metaphorically said as married in option B. Hence, option B is the answer.

Q. 124 Which of the following statements about modern science best captures the theme of the passage? 

A Modern science rests firmly on the platform built by the Greeks. 

B We need to go back to the method of enquiry used by the Greeks to better understand the laws of dynamics. 

C Disciplines like Mathematics and Physics function best when integrated into one. 

D New knowledge about natural phenomena builds on existing knowledge. 

Answer: D 

Explanation: 

The author says that Einstein’s principle is merely an extension of classical Newtonian principle. Option D agrees with this saying that new knowledge about natural phenomena builds on existing knowledge. Hence, option D is the answer. 

Q. 125 The significant implication of Einstein’s special principle of relativity is that 

A absolute velocity was meaningless in the realm of mechanics. 

B Newton’s principle of relativity needs to be modified. 

C there are limits to which experimentation can be used to understand some physical phenomena. 

D it is meaningless to try to understand the distinction between velocity and accelerated or rotational motion. 

Answer: C 

Explanation: 

The author says that “Its SIGNIFICANCE lay in its assertion that absolute Galilean motion or absolute velocity must ever escape all experimental detection.” 

Here, “it” refers to Einstein’s principle. 

The meaning of the sentence is that it is not always possible to experiment. 

Option C gives a similar meaning. Hence, C is the answer. 

 

Instructions 

As you set out for Ithaka 

hope the journey is a long one, 

full of adventure, full of discovery. 

Laistrygonians and Cyclops, 

angry Poseidon – don’t be afraid of them: 

you’ll never find things like that on your way 

as long as you keep your thoughts raised high, 

as long as a rare excitement 

stirs your spirit and your body. 

Laistrygonians and Cyclops, 

wild Poseidon – you won’t encounter them 

unless you bring them along inside your soul, 

unless your soul sets them up in front of you. 

Hope the voyage is a long one, 

may there be many a summer morning when, 

with what pleasure, what joy, 

you come into harbours seen for the first time; 

may you stop at Phoenician trading stations 

to buy fine things, 

mother of pearl and coral, amber and ebony, 

sensual perfume of every kind – 

as many sensual perfumes as you can; 

and may you visit many Egyptian cities 

to gather stores of knowledge from their scholars. 

Keep Ithaka always in your mind. 

Arriving there is what you are destined for. 

But do not hurry the journey at all. 

Better if it lasts for years, 

so you are old by the time you reach the island, 

wealthy with all you have gained on the way, 

not expecting Ithaka to make you rich. 

Ithaka gave you the marvelous journey, 

without her you would not have set out. 

She has nothing left to give you now. 

And if you find her poor, Ithaka won’t have fooled you. 

Wise as you will have become, so full of experience, 

you will have understood by then what these Ithakas mean. 

 

Q. 126 Which of the following best reflects the central theme of this poem? 

A If you don’t have high expectations, you will not be disappointed. 

B Don’t rush to your goal; the journey is what enriches you. 

C The longer the journey the greater the experiences you gather. 

D You cannot reach Ithaka without visiting Egyptian ports. 

Answer: B 

Explanation: 

The four lines “Better if it last for years ………… not expecting Ithaka to make you rich” gives us the central theme of the poem. 

It says that the journey is more important than the goal. 

This meaning is conveyed in option B. 

Hence, option B is the answer. 

 

Q. 127 The poet recommends a long journey. Which of the following is the most comprehensive reason for it? 

A You can gain knowledge as well as sensual experience. 

B You can visit new cities and harbours. 

C You can experience the full range of sensuality. 

D You can buy a variety of fine things. 

Answer: A 

Explanation: 

Option A gives the big picture of why the poet recommends a long journey. 

The remaining options are short-sighted and the poet was not much concerned about them. 

Q. 128 In the poem, Ithaka is a symbol of 

A the divine mother. 

B your inner self. 

C the path to wisdom. 

D life’s distant goal. 

Answer: D 

Explanation: 

The whole poem was about the goal and the journey to reach the goal. 

If we see the lines “Better if it lasts for years …………. not expecting Ithaka to make your rich”, we can see that the poet is comparing Ithaka to life’s distant goal. 

Option D is the answer. 

Q. 129 What does the poet mean by ‘Laistrygonians’ and ‘Cyclops’? 

A Creatures which, along with Poseidon, one finds during a journey. 

B Mythological characters that one should not be afraid of. 

C Intra-personal obstacles that hinder one’s journey. 

D Problems that one has to face to derive the most from one’s journey. 

Answer: C 

Explanation: 

The poet is trying to say that one who pursues the journey to one’s goal must not be afraid of hindrances in that journey. Option C is similar to our conclusion. 

Hence, option C is the answer. 

Q. 130 Which of the following best reflects the tone of the poem? 

A Prescribing. 

B Exhorting. 

C Pleading. 

D Consoling. 

Answer: B 

Explanation: 

The poet says that Ithaka is the reason for one’s journey. 

So, he is trying to be encouraging. 

Among the given options, only exhorting fits the be correct answer. 

Instructions 

For the following questions answer them individually 

Q. 131 Choose the option that conforms most closely to Standard English usage. 

A The running of large businesses consist of getting somebody to make something that somebody else sold to somebody else for more than its cost. 

B The running of a large business consists of getting somebody to make something that somebody else will sell to somebody else for more than it costs. 

C The running of a large business consists of getting somebody to sell something that somebody else made for more than it cost. 

D The running of large businesses consist of getting somebody to make something else that somebody else will sell to somebody else for more than it costs. 

E None of the above 

Answer: B 

Explanation: 

Option a and d are grammatically incorrect as “The running of large business” is singular and hence should be followed by “consists.” Option c doesn’t make any sense as it is contextually incorrect. Option b is the most appropriate.  

Q. 132 Choose the option that conforms most closely to Standard English usage. 

A From the sixteenth century onwards, people started feeling disdainful and self-conscious about their body and its products that led to a heightened focus on emotional and bodily regulations. 

B The heightened focus on controlling the body and emotions comes from disdain and selfconsciousness about the body and its products, found in the sixteenth century. 

C From the sixteenth century onwards, a growing disdain for and self-consciousness about the body and its products took hold, leading to a heightened focus on emotional and bodily regulation. 

D The heightened focus on emotional and bodily regulations started from the sixteenth century onwards, when people felt disdain and self-consciousness about the body and its products. 

Answer: C 

Explanation: 

Option a is incorrect as the usage of “feeling disdainful” is incorrect. Option d is also incorrect due to the usage of “felt disdain.” Option b is having a tense error as there is a present form used in the sentence. 

Q. 133 Choose the option that conforms most closely to Standard English usage. 

A We are forced to fall back on fatalism as an explanation of irrational events. 

B We are forced to falling back on the fatalism as an explanation of irrational events. 

C We are forced to fall back on fatalism as explanations of irrational events. 

D We are forced to fall back to fatalism as an explanation of irrational events. 

Answer: A 

Explanation: 

Option d is incorrect as “to fatalism” is incorrect, option b is incorrect as the usage of “falling back” is incorrect. Option c is incorrect as “falling back on fatalism” is singular and hence the correct usage should be “explanation” rather than “explanations.” 

Q. 134 Choose the option that conforms most closely to Standard English usage. 

A Creativity in any field is regarded not only as valuable for itself but also as a service to the nation. 

B Creativity in any field is not regarded only as valuable on its own, but also as a service to the nation. 

C Creativity, in any field, is not only regarded as valuable, but also as a service to the nation. 

D Creativity in any field is regarded not only as valuable in itself but also as a service to the nation. 

Answer: D 

Explanation: 

The correct usage would be “The creativity is regarded not only as” 

Or in order to infer the correct usage we would break the sentence into 2 parts “Creativity in any field is regarded as valuable in itself and also regarded as a service to the nation”. 

Q. 135 Choose the option that conforms most closely to Standard English usage. 

AIf precision of thought had facilitated precision of behaviour, and if reflection had preceded action, it would be ideal for humans. 

B It would be ideal for humans if reflection preceded action and precision of thought facilitated precision of behaviour. 

C It would be ideal for humans if precedence of reflection was followed by action and precision of thought, by precise behaviour. 

D It would have been ideal for humans, if precise action and behaviour preceded precise reflection. 

Answer: B 

Explanation: 

In this sentence, the ideal situation is satisfactory. “Reflection” should precede action and thought should facilitate behaviour. 

Q. 136 Choose the most logical order of sentences among the given choices to construct a coherent paragraph. 

A. A few months ago I went to Princeton University to see what the young people who are going to be running our country in a few decades are like. 

B. I would go to sleep in my hotel room around midnight each night, and when I awoke, my mailbox would be full of replies—sent at 1:15 a.m., 2:59 a.m., 3:23 a.m. 

C. One senior told me that she went to bed around two and woke up each morning at seven; she could afford that much rest because she had learned to supplement her full day of work by studying in her sleep. 

D. Faculty members gave me the names of a few dozen articulate students, and I sent them e mails, inviting them out to lunch or dinner in small groups. 

E. As she was falling asleep she would recite a math problem or a paper topic to herself; she would then sometimes dream about it, and when she woke up, the problem might be solved. 

A DABCE 

B DACEB 

C ADBCE 

D AECBD 

Answer: C 

Explanation: 

A is the best opening sentence. It introduces the author’s trip to Princeton University. 

This is followed by D, which talks about how the author prepared for his trip by gathering the email ids of a few Princeton students. 

B then follows and describes what happened when the author emailed the students of Princeton. The situation is then explained in sentence C and followed by sentence E. 

Q. 137 Choose the most logical order of sentences among the given choices to construct a coherent paragraph. A. Four days later, Oracle announced its own bid for PeopleSoft, and invited the firm’s board to a discussion. 

B. Furious that his own plans had been endangered, PeopleSoft’s boss, Craig Conway, called Oracle’s offer “diabolical”, and its boss, Larry Ellison, a “sociopath”. 

C. In early June, PeopleSoft said that it would buy J.D. Edwards, a smaller rival. 

D. Moreover, said Mr. Conway, he “could imagine no price nor combination of price and other conditions to recommend accepting the offer.” 

E. On June 12th, PeopleSoft turned Oracle down. 

A CABDE 

B CADBE 

C CEDAB 

D CAEBD 

Answer: A 

Explanation: 

The best opening sentence is C. It gives an introduction to the paragraph saying that Peoplesoft offered to by J.D.Edwards. This is followed by A, which talks about how Oracle wanted to buy Peoplesoft just 4 days later. Statement B then follows, by introducing Peoplesoft’s boss Craig Conway and quoting his statements. D follows B as it continues the statements of Mr.Craig. 

E is the concluding sentence. 

Q. 138 Choose the most logical order of sentences among the given choices to construct a coherent paragraph. 

A. Surrendered, or captured, combatants cannot be incarcerated in razor wire cages; this ‘war’ has a dubious legality. 

B. How can then one characterize a conflict to be waged against a phenomenon as war? C. The phrase ‘war against terror’, which has passed into the common lexicon, is a huge misnomer. 

D. Besides, war has a juridical meaning in international law, which has codified the laws of war, imbuing them with a humanitarian content. 

E.Terror is a phenomenon, not an entity—either State or non-State. 

A ECDBA 

B BECDA 

C EBCAD 

D CEBDA 

Answer: D 

Explanation: 

C opens the paragraph brilliantly. E follows C and explains why the phrase “war on terror” is a misnomer. B continues the explanation started in E. D then follows, and gives another reason why the phrase doesn’t hold water. A concludes the paragraph aptly. 

Q. 139 Choose the most logical order of sentences among the given choices to construct a coherent paragraph. 

A. I am much more intolerant of a human being’s shortcomings than I am of an animal’s, but in this respect I have been lucky, for most of the people I have come across have been charming. 

B. Then you come across the unpleasant human animal—the District Officer who drawled, ‘We chaps are here to help you chaps,’ and then proceeded to be as obstructive as possible. 

C. In these cases of course, the fact that you are an animal collector helps; people always seem delighted to meet someone with such an unusual occupation and go out of their way to assist you. 

D. Fortunately, these types are rare, and the pleasant ones I have met more than compensated for them—but even so, I think I will stick to animals. 

E. When you travel round the world collecting animals you also, of necessity, collect human beings. 

A EACBD 

B ABDCE 

C ECBDA 

D ACBDE 

Answer: A 

Explanation: 

BD is a link. This is because B talks about “the unpleasant human animal” that the author has met and D says that such types are fortunately rare. Similarly, AC is a link. A says that the author is intolerant of human shortcomings, but that he has been lucky in meeting mostly charming people. C explains why this might be the case. E is the best opening sentence. The order of sentences is, therefore, EACBD. 

Q. 140 Choose the most logical order of sentences among the given choices to construct a coherent paragraph. 

A.To avoid this, the QWERTY layout put the keys most likely to be hit in rapid succession on opposite sides. This made the keyboard slow, the story goes, but that was the idea. 

B. A different layout, which had been patented by August Dvorak in 1936, was shown to be much faster. 

C. The QWERTY design (patented by Christopher Sholes in 1868 and sold to Remington in 1873) aimed to solve a mechanical problem of early typewriters. 

D.Yet the Dvorak layout has never been widely adopted, even though (with electric typewriters and then PCs) the anti-jamming rational for QWERTY has been defunct for years. 

E. When certain combinations of keys were struck quickly, the type bars often jammed. 

A BDACE 

B CEABD 

C BCDEA 

D CAEBD 

Answer: B 

Explanation: 

C is the best opening sentence. It says that the Qwerty keyboard was designed for solving a specific problem. E follows and talks about what that problem is. This is followed by A which talks about how the Qwerty design solved this problem. It also talks about the shortcoming of the Qwerty design. This is followed by B which talks about a different design which tried to solve the same problem. 

The last sentence is D, which says the second design was never widely adopted even though, with the advent of electrical typewriters and PCs, the original problem of the typrewriters ceased to be a problem anymore. 

Q. 141 Choose the option in which the usage of the word ‘Bundle’ is Incorrect or Inappropriate. 

A newborn baby was a bundle of joy for the family. 

B Mobile operators are offering a bundle of additional benefits. 

C He made a bundle in the share market. 

D It was sheer luck that brought a bundle of boy-scouts to where I was lying wounded. 

Answer: D 

Explanation: 

Option 1 is correct as “bundle of joy” refers to the baby. Option 2 is correct as bundle refers to “a collection of benefits” wrapped together. Option 3 is correct as “made a bundle” means “earning a lot of money.” Option 4 is incorrect usage. 

Q. 142 Choose the option in which the usage of the word ‘Distinct’ is Incorrect or Inappropriate. 

A He is distinct about what is right and what is wrong. 

B Mars became distinct on the horizon in the month of August. 

C The distinct strains of Ravi’s violin could be heard above the general din. 

D Ghoshbabu’s is a distinct case of water rising above its own level. 

Answer: A 

Explanation: 

Option a is incorrect as the correct usage would be “He is distinctive about what is right and what is wrong” Q. 143 

Choose the option in which the usage of the word ‘Implication’ is Incorrect or Inappropriate. 

A Everyone appreciated the headmaster’s implication in raising flood relief in the village. B This letter will lead to the implication of several industrialists in the market scam. 

C Several members of the audience missed the implication of the minister’s promise. 

D Death, by implication, is the only solution the poem offers the reader. 

Answer: A 

Explanation: 

Option a is incorrect as the correct usage would be “Everyone appreciated the headmaster’s efforts.” “Implication” is the incorrect usage here.  

Q. 144 Choose the option in which the usage of the word ‘Host’ is Incorrect or Inappropriate. 

A If you host the party, who will foot the bill? 

B Kerala’s forests are host to a range of snakes 

C Ranchi will play the host to the next national film festival. 

D A virus has infected the host computer. 

Answer: C 

Explanation: 

The correct usage would be “Ranchi will play host” 

Q. 145 Choose the option in which the usage of the word ‘Sort’ is Incorrect or Inappropriate. 

A What sort of cheese do you use in pizza? 

B Farmers of all sort attended the rally. 

C They serve tea of a sort on these trains. 

D Let’s sort these boys into four groups. 

Answer: B 

Explanation: 

The correct usage would be “Farmers of all sorts” 

Q. 146 Fill in the Blanks: 

The British retailer, M&S, today formally ______ defeat in its attempt to ______ King’s, its US subsidiary, since no potential purchasers were ready to cough up the necessary cash. 

A admitted, acquire 

B conceded, offload 

C announced, dispose 

D ratified, auction 

Answer: B 

Explanation: 

Option c is incorrect as we never dispose subsidiaries. The usage of dispose is flawed. Option d is incorrect as “formally ratified defeat” is incorrect usage. Option a is incorrect as “acquire” is an incorrect usage because no purchasers are required while acquiring. Option b correctly fits the sentence. 

Q. 147 Fill in the Blanks: 

Early ______ of maladjustment to college culture is ______ by the tendency to develop friendship networks outside college which mask signals of maladjustment. 

A treatment, compounded 

B detection, facilitated 

C identification, complicated 

D prevention, helped 

Answer: C 

Explanation: 

The detection of maladjustment to college culture becomes complicated when we develop friendships outside the college. Hence option c is correct. 

Q. 148 Fill in the Blanks: 

The ______ regions of Spain all have unique cultures, but the ______ views within each region make the issue of an acceptable common language of instruction an even more contentious one. 

A different, discrete 

B distinct, disparate 

C divergent, distinct 

D different, competing 

Answer: A 

Explanation: 

The answer to the first blank is “different”. There are always different regions in a country. On the other hand views can be “discrete” or “distinct.” Hence option a is the most appropriate option. 

Q. 149 Fill in the Blanks: 

A growing number of these expert professionals ______ having to train foreigners as the students end up ______ the teachers who have to then unhappily contend with no jobs at all or new jobs with drastically reduced pay packets. 

A resent, replacing 

B resist, challenging 

C welcome, assisting 

D are, supplanting 

Answer: A 

Explanation: 

The second part of the sentence says that the expert professionals will end up with no jobs. Hence replace is the correct word for the second blank and the answer is option A. “Resent” is the correct answer for the first blank. 

Q. 150 Fill in the Blanks: 

Companies that try to improve employees’ performance by ______ rewards encourage negative kinds of behavior instead of ______ a genuine interest in doing the work well. 

A giving, seeking 

B bestowing, discouraging 

C conferring, discrediting 

D withholding, fostering 

Answer: D 

Explanation: 

In order to complement the phrase “in doing the work well” the second blank must have a positive word hence eliminating options B and C. “Seeking” is a wrong usage in this sentence. Author of this sentence is trying to say that the performance is not improved by withholding the rewards. Hence the answer is option D. 

Leave a Reply

×

Hello!

Click one of our representatives below to chat on WhatsApp or send us an email to info@vidhyarthidarpan.com

×